infosys puzzles

1.There is a escalator and 2 persons move down it.A takes 50 steps and B takes
75 steps while the escalator is moving down. Given that the time taken by A to
take 1 step is equal to time taken by B to take 3 steps. Find the no. of steps
in the escalator while it is staionary.

Solution (not sure):
If A
takes 1 step in one second, then B takes 3 steps in one second. If A takes t1
seconds to take 50 steps, then B takes 150 steps in t1 seconds.
For B, to
take 150 steps he requires t1 seconds,
Then to take 75 steps he requires t1/2
seconds.
So now, s1=50, t1 = t1 & s2=75, t2=t1/2
ans= (s1*t2 ~ s2*t1)
/ (t1 ~ t2) which gives 100.
so 100 steps is the answer

2. If 5/2
artists make 5/2 paintings using 5/2 canvases in 5/2 days then how many artists
r required to make 25 paintings using 25 canvases in 25 days?

3. If the
digits of my present age are reversed then i get the age of my son.If 1 year ago
my age was twice as that of my son.Find my present age.
ans. father-73,
son-37

4. There are 6561 balls out of them 1 is heavy.Find the min. no.
of times the balls have to be weighed for finding out the haevy ball.
ans.
8

5. If i walk with 30 miles/hr i reach 1 hour before and if i walk with
20 miles/hr i reach 1 hour late.Find the distance between 2 points and the exact
time of reaching destination is 11 am then find the speed with which it
walks.
ans. 120miles and 24 miles/hr

6. There r four face cards
(J,Q,K,A) all of different types(diamond,club,spade,heart) and some conditions r
given.find the order of cards
ans. king -> jack -> queen ->
ace
heart diamond spade club

7. If A,B,C,D,E r 5 members of a family.4
of them give true statements :
1. E is my mother in law
2. C is my son in
law's brother
3. B is my father's brother
4. A is
my brother's wife

Who made the stmt. and what r the realtions among
them
ans. E
|
A<-->B--C
|
D
<--> denotes
husband-wife
-- denotes brothers

8. The product of 5 different
temperatures is 12.If all of then r integers then find all the
temperatures
ans. -2,-1,1,2,3

9.There r 9 cities numbered 1 to 9.From
how many cities the flight can start so as to reach the city 8 either directly
or indirectly such the path formed is divisible by 3.
eg. 1368-Flights goes through 1-3-6-8.

10. If i do
this puzzle i find it to be hard than the last puzzle that i did before that
after that..............very complex stmt.
Is that puzzle difficult,easy,can't say or depends on the
no. of puzzles

11. Replace each letter by a digit. Each letter must be
represented by the same digit and no beginning letter of a word can be 0.
O N
E
O N E
O N E
O N E
-------
T E N
-------
Ans: 0 =1, N = 8
,E = 2, T = 7.

12. Ann, Boobie, Cathy and Dave are at their monthly
business meeting. Their occupations are author, biologist, chemist and doctor,
but not necessarily in that order. Dave just told the biologist that Cathy was
on her way with doughnuts. Ann is sitting across from the doctor and next to the
chemist. The doctor was thinking that Boobie was a goofy name for parent's to
choose,but didn't say anything. What is each person's occupation?
Ans: Since
Dave spoke to the biologist and Ann sat next to the chemist and across the
doctor, Cathy must be the author and Ann the biologist. The doctor didn't speak,
but David did, so Bobbie is the doctor and Dave the chemist.

13. Sometime
after 10:00 PM a murder took place. A witness claimed that the clock must have
stopped at the time of the shooting. It was later found that the postion of both
the hands were the same but their positions had interchanged. Tell the time of
the shooting (both actual and claimed).
Ans: Time of shooting = 11:54 PM
Claimed Time = 10:59 PM.

14. Next number in the series is 1 , 2 , 4 , 13
, 31 , 112 , ?

Ans: 224.
No number has digits more than 4. All of them
are 1 , 2, 4, 8 , 16 , 32 , 64 converted to numbers in base 5.

15.
Shahrukh speaks truth only in the morning and lies in the afternoon, whereas
Salman speaks truth only in the afternoon. A says that B is Shahrukh. Is it
morning or afternoon and who is A - Shahrukh or Salman?
Ans: Afternoon.A is
Salman.

16. Two trains starting at same time, one from Bangalore to Mysore and other in opposite
direction arrive at their destination 1 hr and 4 hours respectively after
passing each other. How nuch faster is one train from other?
Ans:
Twice.

17. There are 6 volumes of books on a rack kept in order ( ie
vol.1, vol. 2 and so on ). Give the position after the following changes were
noticed. All books have been changed Vol.5 was directly to the right of Vol.2
Vol.4 has Vol.6 to its left and both weren't at Vol.3's place.Vol.1 has Vol.3 on
right and Vol.5 on left. An even numbered volume is at Vol.5's place Find the
order in which the books are kept now.
Ans: 2, 5,1,3,6,4.

18. I bought
a car with a peculiar 5 digit numbered licence plate which on reversing could
still be read. On reversing value is increased by 78633.Whats the original
number if all digits were different?
Ans: Only 0 1 6 8 and 9 can be read
upside down. So on rearranging these digits, we get the answer as
10968.

19. The shape in the sketch below is that of a square attached to
half of a similar square. Divide it into four equal pieces.
Ans: Hint : The
figure can be divided into 12 equal triangles.


20) There are two
balls touching each other circumferencically. The radius of the big ball is 4
times the diameter of the small all. The outer small ball rotates in
anticlockwise direction circumferencically over the bigger one at the rate of 16
rev/sec. The bigger wheel also rotates anticlockwise at N rev/sec. What is 'N'
for the horizontal line from the centre of small wheel always is
horizontal.

21)
1 2 3 4
+ 3 4 5 5
----------
4 6 8 9
- 2 3
4 5
----------
2 3 4 4
+ 1 2 5 4
------------
3 6 9
8
------------
Q) Strike off any digit from each number in seven rows
(need not be at same place) and combine the same operations with 3 digit numbers
to get the same addition. After this strike off another digit from all and add
all the No.s to get the same 2 digit No. perform the same process again with 1
digit No.s. Give the ' no.s in 7 rows at each stage.

22) There is a safe
with a 5 digit No. The 4th digit is 4 greater thansecond digit, while 3rd digit
is 3 less than 2nd digit. The 1st digit is thrice the last digit. There are 3
pairs whose sum is 11. Find the number.
Ans: 65292.

23) there are 2
guards Bal and Pal walking on the side of a wall of a wearhouse(12m X 11m) in
opposite directions. They meet at a point and Bal says to Pal "See you again in
the other side". After a few moments of walking Bal decides to go back for a
smoke but he changes his direction again to his previous one after 10 minutes of
walking in the other(opposite) direction remembering that Pal will be waiting
for to meet. If Bal and Pal walk 8 and 11 feet respectively, how much distance
they would have travelled before meeting again.

24. 13 kigs and 6 libs
can produce 510 tors in 10 hrs, 8 kigs and 14 libs can produce 484 tors in 12
hrs.
Find the rate of production of tors for kigs and libs. Express the
answer in
tors/hr.

25)
xxx)xxxxx(xxx
3xx
-------
xxx
x3x
-------
xxx
3xx
-------

Q)
Find the 5 digit No.
Hint: 5 is used atleast once in the
calculation.

26) A fly is there 1 feet below the ceiling right across a
wall length is 30m at equal distance from both the ends. There is a spider 1
feet above floor right across the long wall eqidistant from both the ends. If
the width of the room is 12m and 12m, what distance is to be travelled by the
spider to catch the fly, if it takes the shortest path.

27) Ramesh sit
around a round table with some other men. He has one rupee more than his right
person and this person in turn has 1 rupee more than the person to his right and
so on, Ramesh decided to give 1 rupee to his right & he in turn 2 rupees to
his right and 3 rupees to his right & so on. This process went on till a
person has 'no money' to give to his right. At this time he has 4 times the
money to his right person. How many men are there along with Ramesh and what is
the money with poorest fellow.

28)Question related to probabilities of
removing the red ball from a basket, given that two balls are removed from the
basket and the other ball is red. The basket contains blue,red,yellow
balls.

29)Venkat has 1boy&2daughters.The product of these children
age is 72.The sum of their ages give the door numberof Venkat.Boy is elder of
three.Can you tell the ages of all the three.

30)L:says all of my other 4
friends have money
M:says that P said that exact one has money
N:says that
L said that precisely two have money
O:says that M said that 3 of others have
money.
P:Land N said that they have money.
All are liers.
Who has money
& who doesn't have?

31)Post man has a data of name surname door
number, pet name of 4 families. But only one is correct for each family. There
are a set of statements & questions.

33) 4 couples have a party.
Depending on the set of statements, find who insulted whom and who is the host
of the party.

34) 5 women given some of their heights (tall,medium,short)
Hair( long, plainted), stards(Black or Brown), sari,2 medium,2-short.Tall->no
sari.Plainted->medium. Answer the combinations.

35) A person has to go
both Northwards & Southwards in search of a job. He decides to go by the
first train he encounters.There are trains for every 15 min both southwards and
northwards. First train towards south is at 6:00 A.M. and that towards North is
at 6:10. If the person arrives at any random time, what is the probability that
he gets into a train towards North.

36) A person has his own coach&.
Whenever he goes to railway station he takes his coach. One day he was supposed
to reach the railway station at 5 O'clock. But he finished his work early and
reached at 3 O'clock. Then he rung up his residence and asked to send the coach
immediately. He came to know that the coach has left just now to the railway
station. He thought that the coach has left just now to the railway station. He
thought that he should not waste his time and started moving towards his
residence at the speed of 3 miles/hr. On the way, he gets the coach and reaches
home at 6 o'clock. How far is his residence from railway station.

37)
Radha,Geeta & Revathi went for a picnic. After a few days they forgot the
date, day and month on which they went to picnic. Radha said that it was on
Thursday, May 8 and Geeta said that it was Thursday May 10. Revathi said Friday
Jun 8. Now one of them told all things wrongly, others one thing wrong and the
last two things wrongly. If April 1st is tuesday, what is the right day, date
and month?


38. There is 66x33m rectangular area. Ram is 11/8 times
faster than Krishna. Both of them started walking at opposite ends and they met
at some point then, Ram said "See you in the other end" Then they continued
walking. After some time Ram thought he will have tea so he turned back walked
back 15 meters then he changed his mind again andcontinued walking. How much
Krishna has traveled by the time they meet?

39. There are 5 burglars and
once went to a bakery to rob it obviously The first guy ate 1/2 of the total
bread and 1/2 of the bread. The second guy ate 1/2 of the remaining and 1/2 of
the bread. The third guy ,fourth guy and fifth guy did the same. After fifth guy
there is no bread left out. How many bread are there?

40. All members
belonging to D are members of A.
All members belonging to E are members of
D.
All members belonging to C are members of both A & D.
Some members
of A does not belong to D. All members belonging to D are members of E. 5
questions are there.

41. Write each statements true or false:-
1. The
sum of the 1st three statements and the 2nd false statement gives the true
statement.
2.The no. of true statements false statement.
3. The sum of 2nd
true statement and 1st false statement gives the first true statement.
4.
There are at most 3 false statements.
5.There is no two consecutive true
statements.

42. There are twelve consecutive flags at an equal interval
of distance. A man passes the 8th flag in 8 seconds. How many more seconds will
he take to pass the remaining 4 flags?

43. A person has to cover the
fixed distance through his horses. There are five horses in the cart. They ran
at the full potential for the 24 hours continuously at constant speed and then
two of the horses ran away to some other direction. So he reached the destination 48 hours behind the schedule. If the
five horses would have run 50 miles more, then the person would have been only
24 hours late. Find the distance of the destination.

44. A boat M leaves
shore A and at the same time boat B leaves shore B. They move across the river.
They met at 500 yards away from A and after that they met 300 yards away from
shore B without halting at shores. Find the distance between the shore A &
B.
45. A person was going through train from Bombay to Pune. After every five
minutes he finds a train coming from opposite direction. Velocity of trains are
equal of either direction. If the person reached Pune in one hour then how many
trains he saw in the journey?

46. Food grains are to be sent to city from
godown. Owner wants to reach the food grains at 11 O' Clock in the city. If a
truck travels at a speed of 30km/hr then he will reach the city one hour
earlier. If the truck travels at a speed of 20km/h then he will reach the city
one hour late. Find the distance between the godown to city. Also with which
speed the truck should travel in order to reach at exactly 11 'O
clock.

47. There are five persons A,B,C,D,E whose birthdays occur at the
consecutive days. Birthday of A is some days or day before C & birthday of B
is exactly the same days or day after E. D is two days older than E. If birth
day of C is on Wednesday then find out the birthdays of other.

48.
Persons say these statements.
A says either Democratic or liberal wins the
elections.
B says Democratic wins.
C says neither democratic nor liberal
wins the election.
Of these only one is wrong. Who wins the
election?

49. Six persons A,B,C,D,E &F went to soldier cinema. There
are six consecutive seats. A sits in the first seat followed by B, followed by C
and so on. If A taken on of the six seats, then B should sit adjacent to A. C
should sit adjacent to A or B. D should sit adjacent to A, B or C and so on. How
many possibilities are there ?

50. Suppose there are four grades A, B, C,
D. (A is the best and D is the worst) 4 persons Jack, Jean, Poul and Lucy wrote
the final exam and made the statements like this:-
1. Jack: If I will get A
then Lucy will get D.
2. Lucy: If I will get C then Jack will get D.
Jack
grade is better than Poul grade.
3. Jean: If Jean doesn't get A then Jack
will not get A.
4. Poul: If Jack get A, then Jean will not get B, Lucy will
get C, I won't either A or B.
If all the above statements are true, then
which person will get which grade?

51. Each man dances with 3 women, Each
women dances with 3 men. Among each pair of men they have exactly two women in
common. Find the no. of men and women.

52. A survey was taken among 100
people to find their preference of watching t.v. programmes. There are 3
channels. Given no of people who watch at least channel 1, at least channel 2,at
least channel 3, no channels at all, at least channels 1 and 3, at least
channels 1 and 2, at least channels 2 and 3. Find the no of people who watched
all three.

53. A bird keeper has got P pigeon, M mynas and S sparrows.
The keeper goes for lunch leaving his assistant to watch the birds. Suppose
p=10, m=5, s=8.
a.) When the bird keeper comes back, the assistant informs
that x birds have escaped. The bird keeper exclaims oh no! all my sparrows are
gone. How many birds flew away.
b.) when the bird keeper come back, the
assistant told him that x birds have escaped. The keeper realised that atleast 2
sparrows have escaped. What is minimum no of birds that can escape.

54.
Select from the five alternatives A,B,C,D,E. At the end of each question, two
conditions will be given. The choices are to filled as follows.
A: If a
definite conclusion can be drawn from condition 1.
B: If a definite
conclusion can be drawn from condition 2.
C: If a definite conclusion can be
drawn from condition 1 and 2.
D: If a definite conclusion can be drawn from
condition 1 or 2.
E: No conclusion can be drawn using both
conditions.

1. Person 1 says N<5
Person 2 says n>5.
Person 3
says 3N>20
Person 4 says 3n>10
Person 5 says N<8.
What is the
value of N?

55. There are N coins on a table. There are two players
A&B. You can take 1or 2 coins at a time. The person who takes the last coin
is the loser. A always starts first.

1. If N=7, then
a) A can always
win by taking two coins in his first chance.
b) B can win only if A takes two
coins in his first chance.
c) B can always win by proper play.
d) none of
the above.

2. A can win by proper play if N is equal to
a) 13 b) 37 c)
22 d) 34 e) 48
Ans: E.

3. B can win by proper play if N is equal
to
a) 25 b)26 c) 32 d) 41 e) none

4. if N<4, can A win by proper
play always?

56. There are 4 parties A,B,C,D. There are 3 people x,y,z.
X-says A or D will win. Y-says A will not win. Z-says B or D will not win. Only
one of them is true. Which party won?

57. 5 persons R,S,T,U,V are
contesting for a medal. Evaluation is over English, Maths, Physics, Chemistry
and Hindi. Toper will get 5 marks, least will get 1 mark. No ties any where. R
get 24 and won the overall medal. V gets first in Chemistry and third in Hindi,
T got consistent scores in 4 subjects. Their final standings where in the
alphabetical order. What was the score of S in Chemistry.

57. There are 3
types of castes, say A,B,C. A- always tells truth, B- always false, C-
alternating.
X says --> Z is of C type, I am of A type.
Y says --> X
is a B type.
Z says --> X is of B type.
Who is of which
type?

58. Persons A and B. Person A picks a random no. from 1 to 1000.
Then person B picks a random no. from 1 to 1000. What is the probability of B
getting no. greater then what A has picked?

59. Three boys and three
girls brought up together. Jim, Jane, Tom, Virgina, Dorthy, XXX. They marry
among themselves to form three couples. Conditions are:-
i) Sum of their ages
would be the same.
ii) Virgina was the oldest.
iii) Jim was dorthy's
brother.
iv) Sum of ages Jane+Jim and Tom+dorthy is same.
Give the three
couples.

60. X^(1/3) - X^(1/9) =60. Solve for X.

61. X Z Y+X Y Z =
Y Z X.
Find the three digits.

62. Two boats start from opposite banks
of river perpendicular to the shore. One is faster then the other. They meet at
720 yards from one of the ends. After reaching opposite ends they rest for
10mins each. After that they start back. This time on the return journey they
meet at 400yards from the other end of the river. Calculate the width of the
river.

63. Basketball Tournament organizers decided that two consecutive
defeats will knock out the team. There are 51 teams participating. What is the
maximum no. of matches that can be played.

64. The Master says to his
grandmaster that me and my three cousins have ages in prime nos. only. Summation
of our ages is 50. Grandmaster who knows the age of the master instantly tells
the ages of the three cousins. Tell the ages of three cousins.( 1 is not
considered as prime no.)

65. There are two families Alens and smiths.
They have two children each. There names are A,B,C,D whose ages are different
and ages are less then or equal to 11. The following conditions are
given:-
i) A's age is three years less then his brother's age .
ii) B is
eldest among the four.
iii) C is half the age of the eldest in Alens
family.
iv) The difference in sum of the ages of Alens children and smiths
children is same as that of five years ago.
Find the ages of all the
children.

66. a,b,c,d,e are having numerical values. There are some
conditions given:-
a) a=c <=== b!=e
b) Difference between a and c as
same as difference between c and b as same as difference between a and d.
c)
cd.
Then find a,b,c,d,e.

67. There are six cards in which it has two
king cards. all cards are turned down and two cards are opened.
a) What is
the possibility to get at least one king.
b) What is the possibility to get
two kings.

68. There are 5 persons a,b,c,d,e and each is wearing a block
or white cap on his head. A person can see the caps of the remaining four but
can't see his own cap. A person wearing white says true and who wears block says
false.
i) a says i see 3 whites and 1 block.
ii) b says i see 4
blocks.
iii) e says i see 4 whites.
iv) c says i see 3 blocks and 1
white.
Now find the caps weared by a,b,c,d and e.

69. There are two
women, kavitha and shamili and two males shyam, aravind who are musicians. Out
of these four one is a pianist, one flutist, violinist and drummer.
i) Across
aravind beats pianist.
ii) Across shyam is not a flutist.
iii) Kavitha's
left is a pianist.
iv) Shamili's left is not a drummer. v) Flutist and
drummer are married.

70. When Arthur is as old as his father Hailey is
now, he shall be 5 times as old as his son Clarke is now. By then, Clarke will
be 8 times older than Arthur is now. The combined ages of Hailey and Arthur are
100 years. How old is Clarke?

71. The seven digits in this subtraction
problem are 0, 1, 2, 3, 4, 5 and 6. Each letter represents the same digit
whenever it occurs.
D A D C B
- E B E G
--------------------
B F E
G
--------------------
What digit is represented by each
letter?

72. The Jones have named their four boys after favorite
relatives; their friends, the Smiths, have done the same thing with their three
boys. One of the families has twin boys. From the following clues, can you
determine the families of all seven children and their ages?
i) Valentine is
4 years older than his twin brothers.
ii) Winston, who is 8, and Benedict are
not brothers. They are each named after a grandfather.
iii) Briscoe is two
years younger than his brother Hamilton, But three years older than
Dewey.
iv) Decatur is 10 years old.
v) Benedict is 3 years younger than
Valentine; they are not related.
vi) The twins are named for
uncles.

73. Motorboat A leaves shore P as B leaves Q; they move across
the lake at a constant speed. They meet first time 600 yards from P. Each
returns from the opposite shore without halting, and they meet 200 yards from.
How long is the lake?

74. On the Island of imperfection there is a
special road, Logic Lane, on which the houses are usually reserved for the more
mathematical inhabitants. Add, Divide and Even live in three different houses on
this road (which has houses numbered from 1-50). One of them is a member of the
Pukka Tribe, who always tell the truth. Another is a member of the Wotta Tribe,
who never tell the truth and the third is a member of the Shalla Tribe, who make
statements which are alternately true and false, or false and true. They make
statements as follows:-
ADD:
1. The number of my house is greater than
that of Divide's.
2. My number is divisible by 4.
3. Even's number differs
by 13 from that of one of the others.
DIVIDE :
1. Add's number is
divisible by 12.
2. My number is 37.
3. Even's number is even.
EVEN
:
1. No one's number is divisible by 10.
2. My number is 30.
3. Add's
number is divisible by 3.
Find to which tribe each of them belongs, and the
number of each of their houses.

75. The names of the inhabitants of
Walkie Talkie Land sound strange to the visitors, and they find it difficult to
pronounce them, due to their length and a few vowel sounds they contain. The
Walkie Talkie guide is discussing the names of four inhabitants –
A,B,C and
D. Their names each contain upto eight syllables, although none of the four
names contain the same number. Two of the names contain no vowel sounds; one
contains one vowel sound; and one contains two vowel sounds. From the Guide's
statements below, determine the number of syllables and vowel sounds in each of
the four Walkie Talkie names:-
i) The one whose name contains two vowel
sounds is not A.
ii) C's name does not contain more than one vowel sound or
fewer than seven syllables.
iii) The name with seven syllables does not
contain exactly one vowel sound.
iv) B and C do not have names with the same
number of vowel sounds.
v) Neither the name with five syllables nor the name
with seven syllables contains more than one vowel sound.
vi) Neither the name
with six syllables, nor the B's name, contains two vowel sounds.

76. Two
identical twins have a very unusual characteristic. One tells nothing but lies
on Mondays, Wednesdays and Fridays, and tells nothing but the truth all other
days. The other tells nothing but lies on Tuesdays, Thursdays and Saturdays, and
tells nothing but the truth all other days. On Sundays both children speak the
truth.

77. According to the information presented, which of the following
conversations will be impossible.
a)Twin A : "Today you are a lier"
Twin B
: "You are telling the truth"
b)Twin A : "Today you are a lier"
Twin B :
"Today I am a truth teller"
c)Twin A : "Tommorow I shall be a lier"
Twin B
: "That's correct"
d)Twin A : "Tommorow you will be a lier"
Twin B :
"Today you are a truthteller"
e)Twin A : "Yesterday we were both
truthtellers"
Twin B : "You are lying".

78. Assume that the twins
followed a different set of rules, so that on a given day both told only the
truth while next day both only lied, alternating days of truth telling and
lying. Under these rules,which of the following conversations would be
possible?
a) Twin A : "Today you are a lier"
Twin B : "That is
correct"
b) Twin A : "Today you are a lier"
Twin B : "That is not
so"
c) Twin A : "Tommorow we will be liers"
Twin B : "Yesterday we were
truthtellers"
d) Twin A : "Tommorow we will be liers"
Twin B : "You are 1
year older than I am"
e) Twin A : "We always tell the truth"
Twin B : "We
some times tell the truth".

79. If the twins are heard saying the
following on the same day, which choice presents a correct statement ?
Twin A
: "It is Sunday Today"
Twin B : "Yesterday was Sunday"
Twin A : "it is
summer season now"
a) it is a summer sunday.
b) it is a summer
monday.
c) it is Monday but not summer.
d) it is Sunday but not
summer.
e) it is impossible to determine whether it is Sunday or
Monday.

80. In the month of october in a year has exactly four mondays
and four fridays, find what day of week wiil be on the 20th of November of that
year.
Ans: 20th November was a wednesday.

81. Six persons A,B,C,D,E
& F went to solider cinima. There are six conseutive seats. A sits in one of
the seats followed by B, followed by C and soon. If a taken one of the six seats
, then B should sit adjacent to A. C should sit adjacent A or B. D should sit
adjacent to A, B,or C and soon. How many possibilities are there?
Ans: 32
ways.

83. In mathematica country 1,2,3,4....,8,9 are nine cities. Cities
which form a no. that is divisible by 3 are connected by air planes. (e.g.
cities 1 & 2 form no. 12 which divisible by 3 then 1 is connected to city
2). Find the total no. of ways you can go to 8 if you are allowed to break the
journies.
Ans: 5.

84. ABCDE are sisters. Each of them gives 4 gifts
and each receives 4 gifts No two sisters give the same combination ( e.g. if A
gives 4 gifts to B then no other sisters can give four to other one.)
(i) B
gives four to A.
(ii) C gives 3 to E.
How much did A,B,C,E give to
D?
Ans: Donor no of gifts A 1 B - C 1 D 2

85. There are some
bulbs,which are numbered from 1 to 100.all the bulbs are in on conditions. The
following operations are performed:-
1. Those bulbs number which are
divisible by 2 are switched OFF.
2. Those bulbs numbered which are divisible
by 3 are switched ON (which are already OFF) and OFF bulbs are switched
ON.
3. Similarly bulbs numbers divisible by 4 are either switched ON or OFF
depending upon there previous condition.

4. This procedure is adopted
till 100th bulb.
At the end there were how many bulbs which were in ON
condition?
Ans: 10 ( only perfect squares ).

86. There are different
numbers related with A,B,C,D,E.such that, AB*CD=EEE. E*CD-AB=CC.
Find
AB*D.
Ans: BE.

87. Find the total no of 10 digits whose sum is
4.

88.Four musician problem(refer GRE BARRONS).

89.GRE BARRONS
problem --> Problem number 25 to 28 page no. 4.

90. A, B, C are 3
girls and there are 770 Apples. For every 4 Apples, A takes,B takes 3. For ever
6 Apples, C takes 7 Apples? Ans: 261:145:303.

91) T, U, V are 3 friends
digging groups in fields. If T & U can complete i groove in 4 days &, U
& V can complete 1 groove in 3 days & V & T can complete in 2 days.
Find how many days each takes to complete 1 groove individually. Ans: 24
days.

92) 4 mathematician has x apples. If he arranges them in rows of 3
one will be left. The same is the case with 5,7,9 apples. But when he arranged
them in rows of 11, non will be left. Find the no. of apples.C h e ta na
S
Ans: 946. (Hint: 11*6 11*11 11*16 11*21 =2E......11*76 =3D946).

93)
H starts running after T reaches 1/5th they must when H reach 1/6th, if H wants
win at what speed H should be run? Note: One circle is there, you show this type
of problem.

94) There are 4 mothers, 4 daughters and the colour of their
dresses, and they are aged 1, 2, 3 & 4. Details of the dresses are given
& then it asked about the remaining dresses.

95) There are 5 levels
of dolls and each of different colors & condition are given. Note: This type
of problem also refer.

96) 5 student A, B, C, D, E. One student knows 5
languages. Like that up to one langauge. Conditions:-
*) Spanish is most
popular langauge.
*) 3 persons knows Porchigese.
*) B & C normally
speak English, but when D gathered, they switched to Spanish because that is
only common between the three.
*) Only langauge common between A, B, E is
French.
*) Only langauge common between C & E is Italian.

97. An
escalator is descending at constant speed. A walks down and takes 50 steps to
reach the bottom. B runs down and takes 90 steps in the same time as A takes 10
steps. How many steps are visible when the escalator is not operating. ANS.
150.

98. Every day a cyclist meets a train at a particular crossing. The
road is straignt before the crossing and both are travelling in the same
direction. Cyclist travels with a speed of 10 Kmph. One day the cyclist comes
late by 25 min. and meets the train 5km before the crossing. What is the speed
of the train.
60 kmph.

99. Five persons muckerjee, misra, iyer, patil
and sharma, all take then first or middle names in the full names. There are 4
persons having first or middle name of kumar, 3 persons with mohan, 2 persons
with dev and 1 anil.
-- Either mukherjee and patil have a first or middle
name of dev or misra and iyer have their first or middle name of dev.
-- Of
mukherkjee and misre, either both of them have a first or middle name of mohan
or neither have a first or middle name of mohan.
-- Either iyer of sharma has
a first or middle name of kumar but not both.
Who has the first or middle
name of anil?
Today is Mukherjee.

101. Two turns have vertain peculiar
characteristics. One of them always lies on Monday, Wednesday, Friday. The other
always lies on Tuesdays, thursdays and saturdays. On the other days they tell
the truth. You are given a conversation.
Person A -- Today is sunday and my
name is anil.
Person B -- Today is tuesday and my name is Bill. What is
today?
Today is tuesday.

102) Which of the following statements can be
deduced from the information presented?
i) If it is Sunday, the twins will
both say so.
ii) If it is not Sunday, one twin will give the correct day and
the other will lie about everything.
iii) On any given day, only one twin
will give his correct name.
a) i only.
b) i and ii only.
c) i and iii
only.
d) ii and iii only.
e) i,ii and iii.

103) If the twins are
heard saying the following on the same day, which choice presents a correct
statement?
Twin A : "It is Sunday Today".
Twin B : "Yesterday was
Sunday".
Twin A : "it is summer season now".

a) It is a summer
sunday.
b) It is a summer monday.
c) It is Monday but not summer.
d) It
is Sunday but not summer.
e) It is impossible to determine whether it is
Sunday or Monday.

104. Logical reasoning tactics practice puzzle
poetry.
1) Henny, Axie, Amie are friends. Conditions:-
a) Herry or Axies
is the oldest.
b)If Axie is the oldest, Amie is the youngest.
Who is the
youngest & who is the oldest?
Ans: Amie is the youngest, Axie is
oldest.

105. There is a robery and four persons are suspected out of them
one is
actual thief, these are the sentences said by each one of them!
A
says D had done
B says A had done
C says i dddnt done
D B lied when he
said that i am thief

Out of these only one man is true remaining are
false
ans C is thef, D is true!

106 How many four digit numbers
divisible by four can be formed using
1, 2, 3, 4; repetitions are not
allowed! ans 6

107 A vender solds two things at same cost 12 RS with one
item at 25%profit and other at 20%loss,by this transaction he made profit or
loss by how much? Ans loss,60paise

108. Conversation between two
employees is as follows:-
EMPLOYEE-1: Hello! Now your experience is twice the
my experience.
EMPLOYEE-2: Exactly two times.
EMPLOYEE-1: But at the last
meet, you said that your experience is thrice of my experience.
EMPLOYEE-2:
That is when we met at 2 years back, your experience is thrice that of
yours.
What is the experience of two employess with the company?
Ans:
EMPLOYEE-1: 4 years EMPLOYEE-2: 8 years.


109 ther are four persons
A,B,C,D and for languages english ,french,german,italian.
conditions
1
only one language is spoken by more than two men
2 A dont know english
3 a
man can speak either french or german but not both

4 all man cannot spek
in a group(no common language)
5 A can mediate when B and C want to speak
with each other
6 each men can speak two languages
ans
A french
italian
B english french
C german italian
D german italian

110.
There are 3 women ,they having three jewells, named diamond emerald, ruby
3
women A,B,C 3 thiefs D,E,F each they had taken one jewel from each of the
women
following conditions
one who had taken diamond is the bachelor and
most dangerous
D 's brother in law E who is less dangerous than the thief who
had stolen emerald
(this is the key from this e had stolen ruby)
D did nt
stolen from B
one more condition is there

111. there were three
suspects for a robbery that happend in a bank, tommy,
joy and bruceEach of
them were saying that I haven't done anything and the
other two has done
it.police found that tommy was lying .who is the thief.
3M
(MARKS).

112. Joe started from bombay towards pune and her friend julie
in opposite
direction.they meet at a point .distance travelled by joe was 1.8
miles
more than that of julie.after spending some both started there
way.
joe reaches in 2 hours while julie in 3.5 hours.Assuming both were
travelling
with constant speed.Wath is the distance between the two
cities.

113. there were five hunters A,B,C,D,E and five animals
A,B,C,D,E. Hunter
having the same name with the animal didn't kill it. Each
hunter has
missed some animal.
A animal was hunt by the hunter whose name
matches with animal hunt by
hunter B.
C animal was hunt by the hunter
whose name matches with animal hunt by
hunter D.
E has hunt C and missed D
.find out animals hunted by A,B,C. 6m.

114. A boy picks up the phone and
asks "Who are you?". The voice from the other side answers "I am your mother's
mother-in-law". What is the relation of the boy with the fellow speaking at the
other end.

115. Imagine a rectangle. Its length = 2*width. A square of 1
inch is cut on all corners so that the remaining portion forms a box when
folded. The volume of the box is _____ cubic inches. Find the original
dimensions of the box.

116. 2 persons are doing part time job in a
company say A and B. THe company is open for all the 7 days of the week. 'A'
works every second day. 'B' works every 3rd day. If 'A'works on first june and
'B' works on second june. Find out the date on which both 'A' and 'B' will work
together.

117. Consider a pile of Diamonds on a table. A thief enters and
steals 1/2 of th e total quanity and then again 2 extra from the remaining.
After some time a second thief enters and steals 1/2 of the remaining+2. Then
3rd thief enters and steals 1/2 of the remaining+2. Then 4th thief enters and
steals 1/2 of the remaining+2. When the 5th one enters he finds 1 diamond on the
table. Find out the total no. of diamonds originally on the table before the 1st
thief entered.

118. Imagine 4 persons A,B,C,D. (It is a strength
determining game). A found it hard, but could pull 'C' and 'D' to his side. AC
and BD pairs on opposite sides found themselves equally balanced. When A and B
exchanged thier positions to form pairs AD and BC, BC pair could win and pull AD
to thier side. Order the 4 persons in Ascending order according to thier
strengths.

119. Consider a beauty contest. 3 persons participate. Their
names are Attractive, Delectable, Fascinating. They are from 3 tribes Pukkas,
Wottas, Summas. Pukkas - Always speak truth. Wottas - Always speak lies. Summas
- Speak truth and lies alternatively. Each of the 3 persons make 2 statements.
The person who speaks truth is the least beautiful. From the statements they
give and the character of the 3 tribal types, find out which person belongs to
which tribe. Also find out the persons in the Ascending order of their
beauty.

120. There are 5 positions-Clerk, Buyer, Cashier, Manager,
Floorwalker. There are 5 persons- Mrs.Allen, Mrs.CLark, Twain, Ewing, Bernett.
Conditions:
1. Clerk and cashier lunch time 11.30.to12.30.
2. Others 12.30
to 1.30.
3. Mrs.Allen and Bernett play durind lunch time.
4. Clerk and
cashier share Bachlor rooms.
5. Ewing and Twain are not in good terms because
one day when Twain retuned early from lunch he saw Ewing already sitting for
lunch and reported about him to the manager. Find out which person holds which
post.

121. There are 8 courses to be handled by faculty in 2 semesters. 4
in 1st semester and 4 in 2nd semester. The candiadates hired for the post are k,
l, m, n, o. The courses are Malvino, Shakespeare, Joyce, Chauncer...........
Some conditions will be given like,
1. L and N handle Shakespeare and
Malvino.
2. M and O handle Malvino and Joyce.

122. A family I know has
several children. Each boy in this family has as many sisters as brothers but
each girl has twice as many brothers as sisters. How many brothers
and
sisters are there?
ans: 4 boys and 3 girls.

123. No. of animals is 11
more than the no. of birds. If the no. of birds were
the no. of animals and
no. of animals were the no. of birds( ie., interchanging no.s
of animals and
birds.), the total no. of legs get reduced by one fifth (1/5).
How many no.
of birds and animals were there?
ans: birds:11,animals:22

124. In a
soap company a soap is manufactured with 11 parts.For making one soap you will
get 1 part as scrap. At the end of the day u have 251 such scraps. From that how
many
soaps can be manufactured? ans: 22 + 2+ 1 = 25.

125. 2 * * |
3
* * | No. 7 does not occur in this
---------------- |
5 * * |
multiplication.
* 4 * |
* * 3 | Find the product.
----------------
|
* * * * * |
-------------
|
--------------------------------------------------
ans 2 8 1
3 2
2
-----
5 6 2
5 6 2 0
8 4 3 0 0
---------
9 0 4 8
2
---------

126. There is a 5digit no. 3 pairs of sum is eleven
each.Last digit is 3 times the first one.
3 rd digit is 3 less than the
second. 4 th digit is 4 more than the second one.
Find the digit.
ans :
25296.

127. There are five thieves, each loot a bakery one after the
other such that
the first one takes 1/2 of the total no. of the breads plus
1/2 of a bread.
Similarly 2nd, 3rd,4th and 5fth also did the same. After the
fifth one no.
of breads remained are 3. Initially how many breads were
there?
ans : 31.


128.There are some chicken in a poultry. They are
fed with corn One sack of corn
will come for 9 days.The farmer decides to
sell some chicken and wanted to hold
12 chicken with him. He cuts the feed by
10% and sack of corn comes for 30 days.
So initially how many chicken are
there?

129.Two people X & Y walk on the wall of a godown in opposite
direction.
They meet at a point on one side and then go ahead. X after
walking for some time,
walks in opposite direction for 15 mtrs.Then again he
turns back and walks
in the original direction. What distance did Y walk
before they met again,
if X walks 11 mtrs by the time Y walks
8
mtrs.

130. Problem from SAKUNTALA DEVI 'PUZZLES TO PUZZLE U'.
Problem
no: 23( Walking back to happiness.)


131. Find a five digit number
subject to following conditions:-
a. It contains 2 prime no digits.
b. 3rd
digit is the lagest.
c. 1st digit = (3 rd digit - 1).
c. Sum of 4th digit
and 5th digit is less than 1st digit.
d. Value of the 5th digit lies between
the value of 1st digit and 2nd digit, 5th digit is one half of the 4th
digit.
Ans: 71842

132. 1, 2 , 3, 4 digits are available. How many
number of 4 digited numbers which are divisible by 4. (There should be no
repetition of digits in the numbers) can be formed?
Ans: 6
numbers.

133. A hill of 440 yards is there. Two competitors JACK and JILL
go up the hill, first JACK reaches the topmost and immediatly starts back and
meet JILL 20 yards from the topmost point. Finally JACK reaches the starting
point 0.5 minutes earler than JILL. Speed while coming down is 1.5 times the
speed of going up. Find the time taken by JACK for whole journey (880
yards)?
Ans: 6.3 minutes.

134. A merchant in the last day sells 2
lamps for Rs.12 price. He finds that he has got 25 % gain on one and 20% lost on
the other. Did he loose or gain overall? If so how much?
Ans: 60 paise
Loss.

135. 4 persons are there caledd JOHN, JACOB, PITER, and WILLIAMS. 4
laungages are there named ENGLISH, ITALIAN, GERMAN, FRENCH. Conditions:-
a.
There is no common language for all.
b. Except one language, no language is
spoken by more than two.
c. One can know either German or FRENCH but not
both.
d. John can't speak ENGLISH But John can act as interpreter between
JACOB and PITER. Jacob knows GERMAN but he can tailk with WILLIAM who doesn't
know a word of GERMAN.
e. No common language between JOHN, PITER, and
WILLIAMS.
Which two languages does each person speaks?

Hint: ITALIAN
IS SPOKEN BY THREE PERSONS (This hint is given in Question paper)

136. 5
couples are there.
MEN: L, M, N, O, P.
WOMEN: S, T, U, V, W.
10 seats
are in one row. Odd numberd seats are reserved for MEN only Like that many
conditions are there. This problem is from GRE-BARRONS BOOK.


137. I
participated in a race.1/5th of those who are before me are equal to 5/6th of
those behind me. What were the total number of contestants in the
race?

138. Find the 3 digit number. Third digit is square root of first
digit. Second digit is sum of first and third digits. Find the
number.

139. This problem is of time and work type. Some A and some B are
able to produce so many tors in so many hours.(for example 10 A and 20 B are
able to produce 30 tors per hour). Like this one more sentence was given. We
have to find out the rate of working of A and B in tors/hour.

140. A and
B play a game of dice between them. The dice consists of colors on their faces
instead of numbers. A wins if both dice show same color. B wins if both dice
show different colors. One dice consists of 1 red and 5 blue. What must be the
color in the faces of other dice.(i.e how many blue and how many red?). Chances
of winning for A and B are even.

141. A girl has 55 marbles. She arranges
them in n rows. The nth row consists of n marbles, the (n-1)th row consists of
(n-1) marbles and so on. What are the number of marbles in nth row?

142.
This question is of analogy type. Some sentences regarding tastes of people to
poetry are given like all who like A's Poem, like the poems of B. Like this 7 or
8 sentences were given. Questions were based on this.

143. This question
is also of analogy type. Four persons are there A,B,C,D. Each of the four
persons own either P,Q,R,S. 10 sentences using if clause were given. We have to
find out which belongs to whom.

144) Every station in N railroad issues
every other station's ticket. Some stations are added. Now they have to issue 46
more tickets. Give the No. of stations after and before added.

145) There
was a race between 3 people. Me, Doug and Anne. When I take 21 steps the
distance covered is equal to Doug's 24 steps and Anne's 28 steps. I take 6 steps
to every 7 steps of Doug and 8 steps of Anne. Who won the race?

146)How
many bearers will an explorer need for a 6 day march if each man can carry the
food stuff of one man for 4 days.

147) Consider the following
statements:
Albert: Dave did it.
Dave: Tony did it.
Gug: I did not do
it.
Tony: Dave lied when he said that i did it.
(a)If only one out of all
above statements is true, who did it?
(b)If only one out of all above
statements is false, who did it?

148) A contribution of Rs. 500 was
raisedfrom 500 people. The fee was as follows:
Men: Rs.3.00 each
Women:
Rs. 2.00 each
Childern: 0.48 each
If number of women is more than number
of men, how many childern are there?

149) Alice and Liu had some berries.
The total of Alice's berries and square of number of berries with Liu is 62. The
total of Liu's berries and square of number of berries with Alice is 176. How
many berries does each of them have?


150) A rope ladder was left down
from a ship. 12 steps of the ladder were exposed at 10:00 am. The queen who was
going to visit the ship, said she would visit at 1:00 pm as she would have to
climb lesser number of steps then. The tide in the sea increases from morning to
afternoon at the rate of 1.2 meters per hour. The distance between any 2 steps
of the ladder is 0.4 mts. How many steps will the queen have to
climb?

151) 5 hunters Doe, Deer, Hare, Boarand Row kill 5 animals. Each
hunter kills an animal that does not correspond to his name. Also each hunter
misses a different animal which again does not correspond to his name.
a) The
Deer is killed by the hunter, known by the name of the animal killed by
Boar.
b) Doe is killed by the hunter, known by name of animal missed by
Hare.
c) The Deer was disappointed to kill only a Hare and missed the
Roe.

152) A local forecast service has accuracy of 2/3 says No rain , and
Meteriological service having accuracy of 4/3 says Rain. if Preference is as no
rain what is the chance of rain?


153) a) 10 1 9 2 8 3 7 4 6 5 5 6 4 7
3 8 2 _ _
b) 2 4 16 512 _
Write the next elements in the
series.

154) A Man is sitting in the last coach of train could not find a
seat, so he starts walking to the front coach ,he walks for 5 min and reaches
front coach. Not finding a seat he walks back to last coach and when he reaches
there, train had completed 5 miles. What is the speed of the train.

155)
The Old car of Mary requires tyres to be changed after each 24000 km. If she
wants to go for 42000 km journey then how many minimum number of tyres she will
need.

156) A coin is so unbalanced that it may come both heads in 2
tosses as it may come tails in a single toss. What is the probabality of getting
a head in a single toss.

157) A pen, pencil and eraser together cost
$1.00. if 2E2P, and 3P>4E then what a single pen will cost?

158) A
local forecast service has accuracy of 2/3 says No rain , and Meteriological
service having accuracy of 4/3 says Rain. if Preference is as no rain what is
the chance of rain?

159) Sherlock holmes thrwated the plan to kidnapp Mrs
mary when they were questioned Mercy and his two associated shipy and rany.when
they were telling the story one of them told one thing wrong and other true, the
other told both true, and the last told both false. examining the following tell
the roles played by each
Mercy:: 1) i wrote the ransome note
2) shipy
broke into the window
rany 1) shipy wrote the ransome note
2) mercy ran
away with the lady
shipy 1)i broke into the window
2)rany wrote the ran
some note.

160) Tom asked kim did you like the stamps? She said yes ,me
and rob too liked them. Kim again said that rob got 3 more than he would have
got, if i would have kept 2 more than, what he got. Tom asked how many u gave
Rob? She replied 2 more than what I got. Tell, how many stamps each rob and kim
got?

161) The virgo club members used to meet every week to play cards.
Each time they used to seat around a round table and for their memory they used
all the possiblecombinations of postions each for a single time only. Can you
tell for how many times they met?
Essays Asked
If you are given a chance
to change a thing in you hometown, what would you change? Give examples why you
want to do so.
Television is creating a communication gap among young
generation.


162. A person needs 6 steps to cover a distance of one
slab. If he increases his foot length (step length) by 3 inches he needs only 5
steps to cover the slabs length. What is the length of the each slab.
Ans: 31
inches.

163. There are 19 red balls and one black ball. Ten balls are put
in one jar and the remaining 10 are put in another jar. What is the possibility
that the black is in the right jar.
Ans: 1/2.

164. There is one lily
in the pond on 1st june. There are two in the pond on 2nd june . There are four
on 3rd june and so on. The pond is full with lilies by the end of the
june.
(i) On which date the pond is half full?
Ans: 29th. --the june has
30 days).
(ii) If we start with 2 lilies on 1st june when will be the pond be
full with lilies.
Ans: 29th June.

165. A lorry starts from Banglore to
Mysore at 6.00 A.M, 7.00 A.M, 8.00 am.....10 pm. Similarly one another starts
from Mysore to Banglore at 6.00 am,7.00 am, 8.00 am.....10.00pm. A lorry takes 9
hours to travel from Banglore to Mysore and vice versa.
(i) A lorry which has
started at 6.00 am will cross how many lorries. Ans: 10.
(ii) A lorry which
had started at 6.00pm will cross how many lorries. Ans: 14.

166. A person
meets a train at a railway station coming daily at a particular time . One day
he is late by 25 minutes, and he meets the train 5 k.m. before the station. If
his speed is 12 kmph, what is the speed of the train.
Ans: 60 kmph.
Refer--Shakuntala Devi Book.

167. A theif steals half the total no of
loaves of bread plus 1/2 loaf from a backery. A second theif steals half the
remaing no of loaves plus 1/2 loaf and so on. After the 5th theif has stolen
there are no more loaves left in the backery. What was the total no of loaves
did the backery have at the biggining.
Ans: 31.

168. A gardener plants
100 meters towards east, next 100 meters towards north,next 100 meters towards
west. 98 meters towards east, 96 meters towards north and 96 meters towards
west, 94 meters towards south. and 94 meters towards east and so on. If a person
walks between the trees what is the total distance travelled by him before he
reaches the center.
Ans: |---------------| | | | | | | | --------|- |
---------------------| -.


169. There are four women and 3 men. They
play bridge one night. Find widow among them. Rules:
(i) wife and husband are
never partners.
(ii) Wife and husand never play more than one game. One night
they played four games as follows:-
1. ------ + ------ vs ------- +
---------
2. ------ + ------ vs ------- + ---------
3. ------ + --*--- vs
------- + ---------
4. ---*-- + ------ vs ------- + ---------
the woman
are marked * above.
Ans: Refer Problem 21. Mind Teasers by
Summers.

170. From a vessel, 1/3rd of the liquid evaporates on the first
day. On the second day 3/4th of the remaining liquid evaporates. What fraction
of the volume is present at the end of the second day.
Ans: 50%.

171.
There is a 4 inch cube painted on all sides. This is cut down into of 1 inch
cubes. What is the no of cubes which have no pointed sides?
Ans:
8.

172. Sam and Mala have a conversation. Sam says I am certainly not
over 40. Mala says I am 38 and you are atleast 5 years older than me. Now, Sam
says you are atleast 39. All the statements by the two are false. How old are
they really?
Ans: Mala = 38 yrs; Sam = 41 yrs.

173. Ram Singh goes to
his office in the city, every day from his suburban house. His driver Gangaram
drops him at the railway station in the morning and picks him up in the evening.
Every evening Ram Singh reaches the station at 5 O' Clock. Gangaram also reaches
at the same time. One day Ram Singh started early from his office and came to
the station at 4 O' Clock. Not wanting to wait for the car he starts walking
home. Mangaram starts at normal time, picks him up on the way and takes him back
house, half an hour early. How much time did Ram Singh walk?

174. In a
railway station, there are two trains going. One in the harbour line and one in
the main line, each having a frequency of 10 minutes. The main line service
starts at 5 o'clock and the harbour line starts at 5.02A.M. A man goes to the
station every day to catch the first train that comes. What is the probability
of the man catching the first train?
Ans: 0.8.

175. A family X went
for a vacation. Unfortunately it rained for 13 days when they were there. But
whenever it rained in the mornings, they had clear afternoons and vice versa. In
all they enjoyed 11 mornings and 12 afternoons. How many days did they stay
there totally?
Ans: 18.

176. Albert and Fernandes have two leg
swimming race. Both start from opposite ends of the pool. On the first leg, the
boys pass each other at 18 m from the deep end of the pool. During the second
leg they pass at 10 m from the shallow end of the pool. Both go at constant
speed but one of them is faster. Each boy rests for 4 seconds at the end of the
first leg. What is the length of the pool?

177. Each alphabet stands for
one digit in the following multiplication.
T H I S
x I S
---------
X
F X X
X X U X
------------
X X N X X
------------
What is the
maximum value T can take?


178. If 1/4 of the time from midnight plus
1/2 of the time from now to midnight is the present time, then What is the
present time? 2. In a 10 digit number, if the 1st digit number is the number of
ones,2nd digit number is the number of twos, and ... so on. 10th digit is the
number of zeroes, then find the number.

179. A train blows a siren one
hour after starting from the station. After that it travels at 3/5th of its
speed it reaches the next station 2 hours behind schedule. If it had a problem
50 miles farther from the previous case,it would have reached 40 minutes sooner.
Find the distance between the two stations.

180. An army 50 miles long
marches at a constant rate. A courier standing at the rear moves forward and
delivers the message to the first person and then turns back and reaches the
rear of the army as the army completes 50 miles. Find the distance travelled by
the courier.


181. Olympic race : 4 contestants : Alan,charlie, Darren
,Brain. There are two races and average is taken to decide the winner. One
person comes at the same position in both the race. Charlie always come before
Darren. Brian comes first once. Alan comes third atleast once. Find the
positions. Alan never comes last. Charlie & Darren comes 2nd atleast
once.


182) There are 6561 number of balls in a bag. Out of which one
is heavy ball. In how many minimum number of weighing you can find the heavy
ball.
Ans: 8.

183) The profit made by a company in one year is enough
to give 6% return on all shares. But as the preffered shares get on return of
7.5%, so the ordinary shares got on return of 5%. If the value of preferd shares
is Rs 4,000000, then what is the value of ordinary shares?
Ans: Rs.
6,000000.

184) There were 50 players playing a game among themselves.
Each player is out of the game when lose 3 matches. What is the number of
matches should be played in order to get the winner.

185) A & B two
places. C & D are two people. C started from A and D started from B. When
they meet each other in the way C traveled 18 m more than D. Then C takes 13 and
half a minute and D takes 24 minutes to reach the other end. What was the
distance between A & B.
Ans: 126.

186) I have been hearing a girl
singing a song for last two score. Song: If seven times five and three times
seven is added to my age it would be as far above six nines and four as the
difference between twice of my age and a score. Given-A score is 20
yrs.

187) A tourist wants to go from A to B. There are four ways to do
this:-
1. To take a wagon. The wagon stops for half an hour at a station in
between a & b and then goes to b.
2. To walk to B. If he leavs A at the
same time the wagon leaves, he will be between by the wagon by 1 mile to reach
B.
3. To walk from A at the same time the wagon leaves from A. He will arrive
at the mid station at the time when the wagon is prepared to leave. He can take
the wagon from there. This will take shortest time.
4. To go on upto the mid
station & to walk from there. He will reach at B 15 minutes before the
wagon.
What is the distance between A & B?.

188) In a train there
is one brakeman, conductor, engineer & fireman. Their names are Art, John,
Tom & Pete given in this order or in reverse order. You have to tell the
occupation of the four, w.r.t. these conditions:-
1. Brakeman has no
relatives.
2. John is older than art.
3. Engineer & fireman are
brothers.
4. John is pete's nephew.
5. Fireman is not conductor's
uncle.
6. Conductor is not engineer's uncle.
Ans: Pete & Tom are
brothers. Tom--Father and John is his
son.
Art--Brakeman.
John--Conductor.
Tom--Engineer.
Pete--Fireman.

189)
There is a 18 strong building and 4 people live in it. They are dentist, lawyer,
accountant, architect. Dentist floor is 5 times the lawyer's floor. Account is
below dentist. If archetect moves two floors up he will be midway between
dentist and account. If architect moves to midway of the building (9th floor)
then he will be middle of dentist & lawyer. Ground floor can be ignored i.e.
floor 0.
Ans: Dentist 15. Accountant 13. Archetect 12. Lawyer 3.

190)
4 ladis, Mrs Margarat, Mrs Price, Mrs Winter & Mrs Ellen went for marketing.
Each went for 2 shops only. Their surnames are lorret, torrey, doris and
marshall. One went to a hardwares shop. Two went to bank. Two went to buchers.
All but dorris went to grocery etc. Who went where?

191). A software
engineer starts from home at 3 pm for evening walk. He walks at a speed of 4
kmph on level ground and then at a speed of 3 kmph on the uphill and then down
the hill at a speed of 6 kmph to the level ground and then at a speed of 4 kmph
to the home at 9 pm. What is the distance on one way?

192). A bag
contains certain number of files. Each file is numbered with one digit of 0 to
9. Suppose the person want to get the number between 1 to 2000 (or 7000 check ).
How many minimum number of files should be present in the bag.
.
193). a +
b + c +d = d + e + f + g = g + h + i =17.
If a = 4, what are the values of d
and g. Each letter taken only one of the digit from 1 to 9.
Ans: a = 4 ,b =
2, c =6, d = 5, e = 3, f = 8, g = 1, h = 7, i = 9.

194. A frog jumps 3 ft
comes back 2ft in a day. In how many day it will come out of 30ft deep
well?
Ans: 28 day.

195. A-B=C
D/E=F
G+H=I
C.F=I
Ans: A=9,
B=5, C=4, F=2.

196. When the actual time pass 1 hr, wall clock is 10 min
behind it. When 1 hr is shown by wall clock, table clock shows 10 min ahead of 1
hr. When table clock shows 1 hr, the alarm clock goes 5 min behind it. When
alarm clock goes 1 hr, wrist watch is 5 min ahead of it. Assuming that all
clocks are correct with actual time at 12 noon, what will be time shown by wrist
watch after 6 hr?
Ans: 5:47:32.5 (n X 60 )50/60 X 70/60 X 55/60 X
65/60.

197. A software engineer just returned from US, has eaten too much
fat & put a lot of weight. Every sunday he starts walking 4 km/hr on level
ground, then up at 3 km\hr, then back down hill at 6km\hr, then again on level
ground at 4km\hr till he reaches his destination. If he returned home at 9 p.m.,
what distance did he covered?
Ans: 24 km.

198. Answer the questions
from facts:-
The members of certain tribe are divided into 3 casts abhor,
dravid amp; magar.
1. An abhor woman can't marry dravid man.
2. A magar
woman can't marry a dravid man.
3. A son takes the caste of his father and a
daughter takes caste of her mother.
5. All marriages except those mentioned,
are not permitted.
6. There are no children born out of a wedlock.

199
There are 2 scales of temp A & B. It was given A varies from 14 to 133 and B
varies from 36 to 87. Find the temperature, when temperature of A is equal to
temp of B.
Ans: 52.5.
Let t=mx+c, c=-70 => m=51/119
a=a.51/119 -70
=> a=52.5.


201. There are 4 married couples, out of which, 3
poeple in a group is needed. But there should not be his or her spouse in the
group. How many groups are possible?
Ans: 32.

202. In the 4 digits
1,2,3,4, how many 4 digited numbers are possible which are divisible by 4?
Repeatations are allowed.
Ans: 64.

203. Two men are going along a
track of rail in the opposite direction. One goods train crossed the first
person in 20 sec. After 10 min the train crossed the other person who is comming
in opposite direction in 18 sec. After the train has passed, when the two
persons will meet?
Ans: Approx. 72 min, check it once.

204. The no. of
children, adults. The no. of adults the no. of boys. The no. of boys no. of
girls. The no. of girls no. of family. Conditions:-
1. No family is without a
child.
2. Every girl has at least one brother and sister.
Ans: c > a
> b > g > f; 9 6 5 4 3.

205. There are 4 boys - Anand, Anandya,
Madan and Murali with nic-names perich, zomie, drummy and madeena not in the
same order. Some conditions.
Ans: Anand : Perich
Anandya : Drummy
Madan
: Zombie
Murali : Madeena

206. There are 2 diamonds, 1 spade and 1
club and 1 ace and also 1 king, 1 jack and 1 ace are arranged in a straight
line.
1. The king is at third place.
2. The left of jack is a heart and
its right is king.
3. No two red colours are in consecutive.
4. The queens
are separated by two cards.
Write the order of which suits (hearts ,clubs)and
names (jacks queens etc.) are aranged?

207. Write each statement as true
or false.
8 Marks
1. The sum of the first three statements and the second
false statement gives the true statement.
2. The no. of true statements >
No. of false statements.
3. The sum of second true statement and first false
statement gives the first true statement.
4. There are atmost 3 false
statements.
5. There are no two consequtive true statements.

208.
There are 3 piles each contains 10, 15, & 20 stones. There are A, B, C, D,
F, G and H persons. One man can catch upto four stones from any pile. The last
man who takes will win. If first A starts next B and so on, who will
win?
Ans: May be F.

209. In a certain department store the position of
Buyer, Cashier, Clerk, Floorwalkar & Manager are held, though not
necessarily respectively, by Evans, Ames, Conroy, Davis amp; Buyer. The cashier
& the manager were roommates in college. The Buyer is bachelor, Evans &
Miss Ames have only business contacts with each other. Mrs. Conroy was greatly
dosappointed when her husband told her that the manager had refuged to give him
a raise. Davis is going to be the best man when the clerk & the cashier are
married. What position does each person held?

210. In a four team
foot-ball tournament, all the teams played each Other in three rounds of matches
as shown in the Table - A. Some of The results of the tournament are shown in
the
Table - B. Using the Clues given below, please fill in the blank columns
in the result Table - B (Goals for & Goals Against?).
Note : Two points
for win, one point for draw & zero points for defeat are awarded.
Clues
:
1. East zone won the tournament despite scoring one less goal than the
runners-up.
2. North zone scored an odd number of goals in their first round
Game.
3. South zone, who failed to score in their final match, were beaten by
a two-goal margin in the first round.
4. East zone lost their match aginst
west zone.
5. All four teams scored goals in the second round matches.
6.
West zone scored the same number of goals against east zone as North zone scored
aginst them.


211. East zone scored four goals in round two
match.
Table - A (Matches Played)
Round 1
North zone vs South
zone
West zone vs east zone.
Round 2
South zone vs West zone
East
zone vs North zone
Round 3
South zone vs East zone
West zone vs North
zone.
Table - B (Results)
Played Won Draw Lost Golas For Goals Against
Points
East Zone 3 - - - ? 3 4
North Zone 3 - - - ? ? 4
West Zone 3 - -
- 4 3 3
South Zone 3 - - - 2 5 1


212. In certain community, there
are thousand married couples. Two thirds of the husbands who are taller than
their wives are also heavier and three quarters of the husbands who are heavier
than their wives are also taller. If there are 120 wives who are taller and
heavier than their husbands, how many husbands are taller and heavier than their
wives?

213. Both the Guptas and Sinhas have two young sons, whose ages
are under Eleven. The names of the boys, whose ages rounded off to the nearest
year are all different, are Rajesh, Praveen, Lalith and Prathap. Taking the ages
of the boys only to the nearest year, the following statements are
true:-
Rajesh is three years younger than his brother is.
Praveen is the
oldest.
Prathap is 5 years older than the younger Sinha's boy.
Lalith is
half as old as one of the Guptha's boys.
The total ages of the boys in each
family differ by the same amount today as they did five years ago.

214. A
long Division
Problem:-
xx)xxxxxxxxx(xxxxxxx
xx
------
xxx
xx
------
xxx
xx
-------
xx
xx
-------
xxx
xxx
-------
-------
In
the complete solution, there are four 5's. Find the missing digits.

215.
Following services are operated by Asian airlines between the two are located in
different countries with different time zones. As it is normally done, the time
shown is the local time - viz IST & TST.
Regular Flight Supersonic
Flight
Arrive Alexandria 17:10 TST 15:40 TST
Depart Alexandria 20:50 TST
22:50 TST
Arrived Rampur 23:40 IST
Is the arrival time of supersonic
flight into Rampur from Alexandria same as the Arrival time of the Regular
flight, assuming each Service - Regular and Supersonic maintains its own
constant speed of flight.


216) A, B, C, D, E related. Four of them
made these statements each:-
i) C is my son-in-law's brother.
ii) B is my
father's brother.
iii) E is my mother-in-law.
iv) A is my brother's
wife.
Who made these statements?


217) A ship is away from the
shore by 180 miles. A plane is travelling at 10 times speed of the ship. How
long from the shore will they meet?

218) A clock showing 6 o'clock takes
30 secs to strike 6 times. How long will it take to strike 12 at
midnight?
Ans: 66 seconds.

219) Only boys aged > 16 wear coats.
Boys aged > 15 go to watch football. Some more statements are given. What can
be said about those who are watching football?

220) There are 3 societies
A, B amp; C having some tractors each. A Gives B and C as many tractors as they
already have. After some days B gives A and C as many tractors as they have.
After some days C gives A and B as many tractors as they have. Finally each has
24 tractors. What is the original No.of tractors each had in the
beginning?
Ans: A - 39. B - 21. C - 12.


221. BE * BE = ACB.
A,
B, C, E are non zero numbers. Find B, E.
Ans: B=1 E=9.

222. A, B, C,
D, E are having numerical values. There are some conditions given:-
a) A=C
<===> B!=E
b) Difference between A and C as same as difference between
C and B as same as difference between A and D.
c) C < A and C >
D,
Then Find A, B, C, D, E.

223. There are six cards, in which, it has
two king cards. All cards are turned down and two cards are opened.
a) What
is the possibility to get at least one king?
b) What is the possibility to
get two kings?

224. A person went to a shop and asked for change for 1.15
paise, but he said that he could not only give change for one rupee but also for
50p, 25p, 10p and 5p. What were the coins he had?
Ans: 1-->50p 4--->10p
1--->25p.

225. There are 3 nurses and they work altogether only once
in a week. No nurse is called to work for 3 consecutive days.
Nurse 1 is off
on tueseday, thursday and sunday. Nurse 2 is off on saturday. Nurse 3 is off on
thursday, sunday.
No two nurses are off more than once a week. Find the day
on which all the 3 nurses were on work.

226. There are 5 persons A, B, C,
D, E and each is wearing a block or white cap on his head. A person can see the
caps of the remaining 4 but can't see his own cap. A person wearing white says
true and who wears black says false.
i) A says I see 3 whites and 1
black.
ii) B says I see 4 blacks.
iii) E says I see 4 whites.
iv) C
says I see 3 blacks and 1 white.
Now Find the caps weared by A, B, C, D and
E.

227. There are two women, Kavitha and Shamili and two males Shyam and
Aravind, who are musicians. Out of these four one is a Pianist, one Flutist,
Violinist and Drummer.
i) Across Aravind beats Pianist.
ii) Across Shyam
is not a Flutist.
iii) Kavitha's left is a Pianist.
iv) Shamili's left is
not a Drummer.
v> Flutist and Drummer are married.

228. 1/3 rd of
the contents of a container evaporated on the 1st day. 3/4 th of the remaining
contents of the container evaporated the second day. What part of the contents
of the container are left at the end of the second day?

229. A man
covered 28 steps in 30 seconds but he decided to move fast and covered 34 steps
in 18 seconds. How many steps are there on the escalator when
stationary?

1. Person1: Most of us are satch J.
Person2: Most of us
are jute S.
Person3: Two of us are satch J.
Person4: Three of us are jute
J.
Person5: I am satch J we have to find who is satch and who is
jute.
Ans: S: Satch J:Jute.

2. Four persons are there to cross a
bridge they have one torch light.
Person A can cross in 1 min.
Person B
can cross in 2 min.
Person C can cross in 5 min.
Person D can cross in 10
min.
They have to cross bridge with in 17 min. At a time only two persons can
cross.
I. A & B ----> 2.
II. A <---- 1.
III. C & D
----> 10.
IV. B <---- 2.
V A & B ---->
2
______________
Total 17.

230. What is the maximum number of
slices can you obtain by cutting a cake with only 4 cuts?
Ans:
16.

231. Three are three boxes. In first box, two white balls. In second
box, 2 black balls. In third box, 1 white & 1 black ball. The lables on the
boxes are not correct. Then you have to open one box and to find the colour of
the balls in all boxes.
Ans: Open the box labled black & white. If white
balls are there then the box labled with white balls contain black balls and
labled with black balls contain one black and one white ball and vice versa, if
two black balls are there.

232. There is a 4 inch cube painted on all
sides. This is cut into number of 1 inch cubes. What is the number of cubes
which have no painted sides?

233. Sam and Mala have a conversation. Sam
says I am certainly not over 40. Mala says I am 38 and you are atleast 5 years
older than me. Now Sam says you are atleast 39. All the statements by the two
are false. How old are they realy?

234. Ram singh goes to his office in
the city every day from his suburban house. His driver Mangaram drops him at the
railway station in the morning and picks him up in the evening. Every evening
Ram singh reaches the station at 5 o'clock. Mangaram also reaches at the same
time. One day Ram singh started early from his office and came to the station at
4 o'clock. Not wanting to wait for the car he starts walking home. Mangaram
starts at normal time, picks him up on the way and takes him back house, half an
hour early. How much time did Ram singh walked?

235. Some people went for
vaction. Unfortunately it rained for 13 days when they were there. But whenever
it rained in the morning, they had clean afternood and vice versa. In all they
enjoyed 11 morning and 12 afternoons. How many days did they stay there
totally?


236. Geoffrey, Hallmann and Molly attend and interview and
give three different statements each.
To make it a little complex. Out of the
three statements made by each one, one is false.
Geoffrey says: I am 22 -
false
Hallmann is elder than me by one year.
Molly is 25

Hallmann
says: I am not the youngest -
Geoffrey is 2 years younger than me
Molly is
younger than me by one year - false

Molly says: Geoffrey is 23
I am a
year younger than Geoffrey
(I don't remember this statement but it is
false.)

Answer:
Goeffrey is 22 as Goef states,
And Goef== 23 as
Molly states.
Both cannot be true.
Either one has to be true or either one
has to be false.
Start deriving from this point and u get,

Geoffrey
Hallmann Molly
Age: 23 25 22


237. There is a cube, which has to be
inscribed with the following pair
of numbers on opposite sides.1 and 6, 2 and
4,3 and 5.
How many different ways can it be done?

238. There's an
electric wire running 1 km from the side of a building.
The number of poles
in between them is placed in an interval of
distance between each
other.
If one pole is removed then the distance between each pole becomes 1
2/3 meters.
Find out how many poles were kept.

239. The time taken to
travel in train from Town A to Town B is 5 hours.
There are trains starting
from both towns at an interval of 1 hour.
How many trains meet in 1
trip?
Ans : 10 trains check it as trains come from both sides every
hour.

240. Shadow went to an Isle where the natives lie and the visitors
speak truth.
Shadow saw a salesman and wanted to know whether he was a native
or a visitor.
He did not pose a question directly but asked him indirect
instead.
Shadow saw a woman and asked the salesman,"Is that a NATIVE
or
VISITOR?" .For which the salesman replied," She is a visitor".
Is the
salesman a Native or a Visitor?
Ans :
Since Shadow himself saw him/her as
a woman and asked the salesman.
The Salesman replied ,"SHE " by which he
speaks truth and is a Visitor.
Salesman is a Visitor.


241. Three
friends divided some bullets equally.
After all of them shot 4 bullets the
total number of bullets remaining is equal to the bullets each had after
division.Find the original number divided.

Ans: 18 (2
marks)

Initially . x x x
Now x-4 x-4 x-4
Equation is 3x-12 =
x



242. There are 3 societies A, B, C.
A lent cars to B and C
as many as they had already.
After some time B gave as many tractors to A and
C as many as they have.
After sometime c did the same thing. At the end of
this transaction each one of them had 24.
Find the cars each orginally
had.
Ans: A had 39 cars, B had 21 cars & C had 12 cars

243. The
Bulls, Pacers, Lakers and Jazz ran for a contest.
Anup, Sujit, John made the
following statements regarding results.
Anup said either Bulls or Jazz will
definitely win
Sujit said he is confident that Bulls will not win
John
said he is confident that neither Jazz nor Lakers will win
When the result
cameit was found that only one of the above three had made a correct
statement.
Who has made the correct statement and who has won the
contest.
Ans: Sujith; Lakers

244. Five people A ,B ,C ,D ,E are
related to each other.
Four of them make one true statement each as
follows.

(i) B is my father's brother.
(ii) E is my
mother-in-law.
(iii)C is my son-in-law's brother
(iv)A is my brother's
wife.

Ans: (i) D (ii) B (iii) E (iv) C

245. Some statements are
given below:

L says all of my other four friends have money
M says
that P said that exactly one among them has money
N says that L said that
precisely two among them have money
O says that M said that three of the
others have money
P, L and N said that they have money
All the above
statement are false..
Who has money & who doesn't have any
money?

246. Fifty minutes ago if it was four times as many minutes past
three o'clock,how many minutes is it to six o'clock?
Ans: Twenty six
minutes.

247. A hotel has 10 storeys.Which floor is above the floor below
the floor, below the floor above the floor, below the floor above the
fifth.
Ans: The sixth floor.

248. Seven members sat around a table for
three days for a conference.
The member's names were Abhishek, Amol, Ankur,
Anurag,Bhuwan ,Vasu and Vikram.
The meetings were chaired by Vikram.
On
the first evening members sat around the table alphabetically.
On the
following two nights, Vikram arranged the seatings so that he could have
Abhishek as near to him as possible and abesent minded Vasu as far away as he
could.http://www.ChetanaS.com
On no evening did any person have sitting next
to him a person who had previously been his neighbour. How did Vikram manage to
seat everybody to the best advantage on the second and third
evenings?

Ans:
Second evening:Vikram,Ankur,Abhishek,Amol,Vasu,Anurag
and Bhuwan.
Third evening
:Vikram,Anurag,Abhishek,Vasu,Bhuwan,Ankur,Amol.


249. Two trains start
from stations A and B spaced 50 kms apart at the same time and speed.
As the
trains start, a bird flies from one train towards the other and on reaching the
second train, it flies back to the
first train.This is repeated till the
trains collide.
If the speed of the trains is 25 km/h and that of the bird is
100km/h.
How much did the bird travel till the collision.
Ans: 100
kms.

250. Four prisoners escape from a prison.
The prisoners, Mr East,
Mr West, Mr South, Mr North head towards different directions after escaping.The
following information of their escape was supplied:

The escape routes
were The North Road, South Road, East Road and West Road.
None of the
prisoners took the road which was their namesake.
Mr.East did not take the
South Road
Mr.West did not the South Road.
The West Road was not taken by
Mr.East
What road did each of the prisoners take to make their
escape?

Ans: Mr.East took the North Road
Mr.West took the East
Road
Mr.North took the South Road
Mr.South took the West Road.

251.
Complete the series:
5, 20, 24, 6, 2, 8, ?

Ans: 12 (as 5*4=20,
20+4=24, 24/4=6, 6-4=2, 2*4=8, 8+4=12).

252) A soldier looses his way in
a thick jungle. At random he walks from his camp but mathematically in an
interesting fashion. First he walks one mile East then half mile to North. Then
1/4 mile to West, then 1/8 mile to South and so on making a loop.
Finally how
far he is from his camp and in which direction.

Ans: Distance travelled
in north and south directions
1/2 - 1/8 + 1/32 - 1/128 + 1/512 - and so
on
= 1/2/((1-(-1/4))
Similarly in east and west directions
1- 1/4 +
1/16 - 1/64 + 1/256 - and so on
= 1/(( 1- ( - 1/4))
Add both the
answers


253) How can 1000000000 be written as a product of two
factors neither of them containing zeros
Ans: 2 power 9 x 5 power
9

254) Conversation between two mathematcians:
First : I have three
childern. The product of their ages is 36.
If you sum their ages, it is
exactly same as my neighbour's door number on my left.
The second
mathematician verfies the door number and says that it is not
sufficient.
Then the first says " Ok one more clue is that my youngest is
really the youngest". Immmediately the second mathematician answers . Can you
answer the question asked by the first mathematician?
What are the childeren
ages? Ans 1,6 and 6

255) Light glows for every 13 seconds . How many
times did it glow between 1:57:58 and 3:20:47 am.
Ans : 383 + 1 =
384

256) 500 men are arranged in an array of 10 rows and 50 columns
according to their heights.
Tallest among each row of all are asked to fall
out.
And the shortest among them is A.
Similarly after resuming that to
their original podsitions that the shortest among each column are asked to fall
out.
And the tallest among them is B .
Now who is taller among A and B
?
Ans A

257) A person with some money spends1/3 for cloths, 1/5 of the
remaining for food and 1/4 of the remaining for travel.
He is left with Rs
100/- . How much did he have with him in the begining ?
Ans: Rs
250/-

258) There are six boxes containing 5 , 7 , 14 , 16 , 18 , 29 balls
of either red or blue in colour.
Some boxes contain only red balls and others
contain only blue.One sales man sold one box
out of them and then he says " I
have the same number of red balls left out as that of blue ".
Which box is
the one he solds out ?
Ans: Total no of balls = 89 and (89-29 /2) = 60/2 =
30
and also 14 + 16 = 5 + 7 + 18 = 30

260) Grass in lawn grows equally
thick and in a uniform rate.
It takes 24 days for 70 cows and 60 days for 30
cows to eat the whole of the grass.
How many cows are needed to eat the grass
in 96 days.?
Ans : 20
g - grass at the beginning
r - rate at which
grass grows, per day
y - rate at which one cow eats grass, per day
n - no
of cows to eat the grass in 96 days
g + 24*r = 70 * 24 * y
g + 60*r = 30 *
60 * y
g + 96*r = n * 96 * y
Solving, n = 20.

261)There r some bees
in a garden..1/5th of them went to a particular
flower,1/3rd went to another
flower,3 times the difference of the
above two went to third flower..n one
was remaining n it was roaming
around..how many bees were there? (3
marks)
Ans:15

262)there was a community in which there were 1000
couples.In that
2/3rd of men who r taller r also heavier n 3/4th of the men
who r
heavier r also taller n there were 120 women who were both heavier
n
taller than men.So how many men r both taller n heavier than
men?

263)A man drives with constant speed..n he after some time he sees
a
milestone with 2-digits..then he travels for an hr n sees the same
2
digits in reverse order..n then after an hr he sees that the
milestone
has the same 2 digits with a 0 between them..so whats the
man
speed?
Ans:45km/hr

264)There were 2 systems A n B.14 degrees in A is
equivalent to 36 in
system B.and 133 in A is equivalent to 87 in B.now what
is the
temperature where they both r equal?
Ans:51.25 (conversion
A=(7/3)B-70)

265) X Y Z X Y Z
A B + A B -
------------
-------------
C D E F B G A

find X,Y,Z,G

266)A women buys some
shoestrips n then 4 times of that she buys
packet pins n then 8 times of
shoestrips she buys handkerchiefs..n
she has a bill of Rs3.24..n she pays for
each article as many paise
as there r articles(of tht particular item).Now
whats the number of
handkerchiefs?
Ans:16

267)Ms.Anitha got her
salary n she spent half of it in shopping n gave
1RS to a beggar.After that
1/2 of the remaining money she spends in
a hotel n she gives Rs.2/- as a tip
to waiter.n then 1/2 of the
remaining she spends again n she gives 3 RS as
charity..n after that
finally she is left with Rs 1/- when she comes
out.Whats the actual
money she had?
Rs 42/- (3 marks)


268. A
person is cycling in a circular track.At some point he notices
that 1/5 of
people in front of him and 5/6 of people together
condtitute the total no. of
cyclists.Find the total no. of cyclists.
Ans:31

269.Trains leave from
New York to Washington every hour on the
hour(1:00,2:00....).Trains leave
from Washington to New York every
hour on the hour and half
hour(1:00,1:30,2:0,2:30....).It takes a train 5
hrs to complete its journey
from Washington to New York as well as
from new york to washington.A train
leaves from new york to washington. Find
out how many trains it will meet
before it reaches washington.
Ans:19 or 21(not sure).

270) A pen,
pencil and eraser together cost $1.00. if 2E2P, and P>4E then what a single
pen will cost?

271. In a class there are less than 500 students . when it
is divided by 3 it gives a whole number. similarly when it is divided by 4,5 or
7 gives a whole number.find the no. of students in the class.
ans:
420

272. Uncle reuben and aunt cynthia came to town to shop reuben bought
a suit and hat for $15
cynthia paid as much as for her hat as reuben did for
his suit then she spent the rest of their
money for a new dress on the way
home cynthia called reuben's attention to the fact that his hat
cost $1 more
than her dress then she added if we had divided our hat money differently so
that
we bought different hats mine costing 1 and 1/2 time cost if yours then
we each would have spent
the same amount of money in that case said uncle
reuben "how much would my hat have cost"
Ans : Uncles hat costs $6.4 (total
money was $29)


273. Four family names are given and their frends name
are given but not in order.( u hav to find
which frend belongs to which
family) Each frend prepares salad using 3 different fruits . they are
given
apple, cherry, grape, banana., no two frens uses the same
combination.
Various conditions are given and 4 ques asked.
Which fruit
did mandy didn't use?
Who is flures friend?
Name the fruits common btwn
Erica and stacy?

274.Here is a simple mathematical puzzle set by
Longfellow in his own flowery, poetical language.
If 1/5th of a hive of bees
flew to the badamba flower, 1/3rd flew to the slandbara, 3 times
the
difference of these two numbers flew to an arbour, ad one bee continued
to fly about, attracted on each side by the fragrant ketaki and malati, what was
the total number of bees?
Ans: 15

275. (don't remember the exact
question)a man while sorting files picks up file number one,misses
one, picks
up file no. 2,mises two files,and so on...after that he found that he picked up
5% of the
files. How many files were there?
Ans:39

276. A man walks
at 4 km/hr on plain, then at 3 km/hr uphill and then returns through the same
road at 6 km/hr downhill and at 4 km/hr on the plain. It takes altogether 6
hours. So what distance he covered in one way?
Ans: 12 km
Solution: Let
plain road = x km
And hill road = y km
? x/4 + y/3 + y/6 + x/4 = 6
?
x/2 + y/2 = 6
? x + y =12

277. There are some men and some lodges for
which the
following conditions hold true –
i) Each lodge is represented by
exactly 3 men .
ii) Each man is associated with exactly 2 lodges.
iii) Any
pair of lodge has only one man in common.
How many men and how many lodges
were there?

Ans: 6 men and 4 lodges.
Solution:
i) L1 is represented
by M1, M2, M3. L2 is represented by M3, M4, M5. L3 is represented by M1, M4, M6.
L4 is represented by M2, M5, M6.
ii) M1 is associated with L1,L3. M2 is
associated with L1,L4. M3 is associated with L1,L2. M4 is associated with L2,L3.
M5 is associated with L2,L4. M6 is associated with L3,L4.
iii) Common between
L1 and L2 is only M3. Common between L1 and L3 is only M1. Common between L1 and
L4 is only M2. Common between L2 and L3 is only M4. Common between L2 and L4 is
only M5. Common between L3 and L4 is only M6.
i.e all the given conditions
are satisfied.

278. A person sells 2 items for Rs. 12 each. For one
he
profits 25% and for the other he losses 20%. Altogether did he loss or
gain? And by how much?

Ans: He losses by 60 paise.
Solution: Cost
price of the item for which he losses = Rs.
12*100/80 = Rs. 15.
Cost price
of the item for which he Gains = Rs.
12*100/125 = Rs 9.6.
? Total cost
price is Rs. 24.6 and total sell price is
Rs. 24.
So altogether he losses
by Rs. 0.6.

279. My father’s age was x in the year x². I am obviously
talking about 20th century. In which year was my father born?

Ans: In the
year 1892.
Solution: x = 44 as 44² = 1936 and the square of any
other
integer will not fall in between 1900 and 2000. i.e my
father was 44
in the year 1936. ? my father was
born in the year 1892.

280.A man was
going by cycle. After going 2/3rd of total distance the cycle broke down and he
had to complete the journey on foot. At the end he found that he walked twice as
long as he was on cycle. How many times the speed of the cycle is as the speed
of walking?

Ans: 4 times.
Solution: Let the distance be z km, speed of
cycle be x km/hr
and walking speed be y km/hr.
Then he covered 2z/3 km by
cycle in 2z/3x hr and
covered z/3 km on foot in z/3y hr.
? according to
the question 2*2z/3x = z/3y ? x/y = 4.

281.In a badminton tournament a
team is eliminated from the tournament if it losses 2 games. If there are 51
teams then what is the maximum number of games required to select the
champion?

Ans: 101
Solution: To eliminate the 50 teams 50*2 = 100
games are
required. And the champion team may lose in at
most 1 game. ?
Max. no. of games required = 101.

282. There are 3 tribes in a Island.
Sororean who always speak truth, Nororean who always speak false and Midorean
who speak truth and false alternately in either order. From the statements given
by A,B and C (who belong to 3 different tribes), three persons from the island,
identify who belongs to which tribe.

A : C is Sororean.
B is
Midorean.

B : A is Nororean.
C is Midorean.

C : A is
Midorean.
I am Sororean.

Ans:
A B C
Midorean Nororean
Sororean
Solution: Think logically.

283. There are 5 persons A,B,C,D,E
. All of them have different occupations and none of them are of same age. There
professions are Consultant, Planner, Engineer, Nutritionist and Technician
definitely not in the same order. From the following conditions determine whose
profession is what?

i) The consultant is the oldest among them.
ii)
The Technician is not younger than the Planner and the Planner is not younger
than the Nutritionist.
iii) D is not as old as A and also not as young as B,
who is not as old as the Engineer and also not as young as C.
iv) C is not
the youngest among them.

Ans: Names are given in descending order of
their ages.
Consultant Engineer Technician Planner Nutritionist
A D B C
E
Solution: Think logically.

284. 4 persons, Watts, Roger, O’neil and
Smith, were eating in a restaurant while Smith was killed by poisoning. During
investigation the other 3 persons gave the following statements. They gave one
false statement each.

Watts : i) I didn’t do it.
ii) O’neil sat beside
me.
iii) We had our usual waiter.

Roger : i) Smith was across the
table from me.
ii) We had a new waiter.
iii) The waiter didn’t do
it.

O’neil : i) Roger didn’t do it.
ii) The waiter has poisoned
Smith.
iii) Watts lied when he said that we had our
usual
waiter.

One among these three persons and the waiter murdered
Smith. Who is the murderer?

Ans : O’neil is the
murderer.

Solution: Think logically.
I have given the false statements
in red and italics.

285 There is a 4 inch cube painted on all sides. This
is cut down into of 1 inch cubes. What is the no of
cubes which have no
pointed sides.
Ans: 8 [(n-2)*(n-2)*(n-2)] where n is side length of the
cube

286. Find the values of each of the alphabets.
N O O N
S O O
N
+ M O O N
J U N E
Ans: 9326

287. If a clock takes 7seconds to
strike 7, how long will the same clock take to strike 10?

Ans: The clock
strikes for the first time at the start and takes 7 seconds for
6
intervals-thus for one interval time taken=7/6. Therefore, for 10 seconds
there are 9 intervals and
time taken is 9*7/6=10 and 1/2 seconds.

288.
Fifty minutes ago if it was four times as many
minutes past three o'clock,
how many minutes is it to
six o'clock?
Ans: Twenty six
minutes.

289. Everyday in his business a merchant had to weigh
amounts
from 1 kg to 121kgs, to the nearest kg.
What are the minimum number of weight
required
and how heavy should they be?
Ans: .The minimum number is 5 and
they
should weigh 1,3,9,27 and 81kgs.


290. Ram, Shyam and Gumnaam
are friends. Ram is a widower and lives alone and his sister
takes care of
him. Shyam is a bachelor and his niece cooks his food
and looks after his
house.Gumnaam is married to Gita and lives in large
house in the same
town.Gita gives the idea that all of them could stay
together in the house
and share monthly expensesequally.
During their first month of
livingtogether, each person contributed Rs.25. At the end of
the month, it
was found that Rs 92 was the expense sothe remaining amount was distributed
equally among
everyone. The distribution was such that everyone received a
whole number of Rupees. How much did each
person receive?Ans. Rs 2 (Hint:
Ram's sister,Shyam's niece and Gumnaam's wife are the same person)

291.
Sam and Mala have a conversation.

• Sam says I am certainly not over
40
• Mala says I am 38 and you are at least 5 years olderthan me
• Now Sam
says you are at least 39
All the statements by the two are false. How old are
they really?
Ans: Mala = 38 yrs, Sam = 41 yrs.

292. Grass in lawn
grows equally thick and in a uniform rate. It takes 24 days for 70 cows and 60
days
for 30 cows to eat the whole of the grass. How many cows are needed to
eat the grass in 96 days.?
Ans : 20
[Hint: g - grass at the beginning r
-rate at which grass grows, per day y - rate at which
one cow eats grass, per
day n - no of cows to eat the grass in 96 days
g + 24*r = 70 * 24* y g +
60*r= 30 * 60 * y g + 96*r = n * 96* y, Solving, n = 20. ]

293. Three
criminals were arrested for shop lifting. However, when interrogated only one
told the truth in
both his statements, while the other two each told one true
statement and one lie. The statements were:
• ALBERT :(a) Chander passed the
merchandise. (b)Bruce created the diversion.
• BRUCE :(a) Albert passed the
merchandise. (b)I created the diversion.
• CLIVE :(a) I took the goods out of
the shop. (b) Bruce passed them over.

Ans: Albert passed the goods.
Bruce
created the diversion. Clive took the goods out of
the
shop.

294. There N stations on a railroad. After adding X stations
on the rail route 46 additional tickets have
to be printed. Find N and
X.

Ans. x=2 and N=11 ( Let initially,N(N-1) = t; After adding,
(N+X)(N+X-1) = t+46;Trail and error method )

295. Complete the Table
given below:
Three football teams are there. Given below is
the group
table. Fill in the x's
Played Won Lost Draw Goals For Goals Against
A 2 2
x x x 1
B 2 x x 1 2 4
C 2 x x x 3 7

Ans: The filled table is given
below

Played Won Lost Draw Goals For Goals Against
A 2 2 0 0 7 1
B
2 0 1 1 2 4
C 2 0 1 1 3 7

297. Four prisoners escape from a prison.
The prisoners, Mr. East, Mr. West, Mr. South, Mr. North
head towards
different directions after escaping. The following information of their escape
was supplied:
• The escape routes were The North Road, South Road,East Road
and West Road.
• None of the prisoners took the road which was
theirnamesake.
• Mr. East did not take the South Road
• Mr. West did not
the South Road.
• The West Road was not taken by Mr. East
What road did
each of the prisoners take to make
their escape?

Ans: Mr. East took
the North Road
Mr. West took the East Road
Mr. North took the South
Road
Mr. South took the West Road.

298. A hotel has two wings, the
east wing and the west wing. Some east wing rooms but not all have an
ocean
view.All west wing rooms have a harbor view. The charge for all rooms
is identical, except as follows :

• Extra charge for all harbor view
rooms on or above
the 3rd floor
• Extra charge for all ocean view rooms
except those
without balcony
• Extra charge for some harbor rooms on the
first two
floor & some east wing rooms without ocean view but
having
kitchen facilities.

Which of the following cannot be determined on
the
basis of the information given:
I. Whether there are any rooms without
a
balcony for which an extra charge is imposed.
II. Whether any room
without a kitchen or
a view involves an extra charge.
III. Whether two
extra charges are imposed
for any room.
(A) I only (B) II only
(C) III
only (D) II and III only
(E) I, II and III
Ans: (A)


299. A ship
went on a voyage. After it had traveled 180 miles a plane started with 10 times
the speed of the
ship. Find the distance when they meet from starting
point.

Ans: 200miles. ( Distance traveled by plane = 1/10 distance
traveled by ship + 180 )

301. Father's age is three years more than three
times the son's age. After three years, father's age will be
ten years more
than twice the son's age. What is the father's present age?
Ans: 33
years.

302. Light glows for every 13 seconds . How many times did it glow
between 1:57:58 and 3:20:47 am.

Ans : 383 + 1 = 384


303. There
are 20 poles with a constant distance between each pole. A car takes 24 second
to reach the
12th pole.How much will it take to reach the last pole.
Ans:
41.45 seconds (Let thedistance between two poles = x, Hence 11x:24::19x:?
)

304. A man collects cigarette stubs and makes one full cigarette with
every 8 stubs. If he gets 64 stubs how
many full cigarettes can he smoke.
Ans: 8+1=9

305. The minute and the hour hand of a watch meet every 65
minutes. How much does the watch lose or gain time
and by how much?
Ans:
Gains; 5/11 minutes

306. A survey was taken among 100 people to find
their preference of watching T. V. programs. There are 3
channels. Given the
no of people who watch
• at least channel 1
• at least channel 2
• at
least channel 3
• no channels at all
• at least channels 1and 3
• at
least channels 1 and 2
• at least channels 2 and 3
Find the no of people
who watched all three.

307. Some statements are given below:

• L
says all of my other four friends have money
• M says that P said that
exactly one among them has
money
• N says that L said that precisely two
among them
have money
• O says that M said that three of the others
have
money
• P, L and N said that they have money
All the above
statement are false. Who has money & who doesn't have any money?

308.
500 men are arranged in an array of 10 rows and 50 columns according to their
heights. Tallest among each
row of all are asked to fall out. And the
shortest among them is A. Similarly after resuming that to
their original
positions that the shortest among each column are asked to fall out. And the
tallest among
them is B. Now who is taller among A and B ?
Ans.
A

309. Mr. Mathurs jewels have been stolen from his bank
locker. The
bank has lockers of 12 people which are
arranged in an array of 3 rows and 4
columns like:
1 2 3 4
5 6 7 8
9 10 11 12
• The locker belonging to
JONES was to the right of
BLACK'S locker and directly above MILLAR'S.

BOOTH'S locker was directly above MILLAR'S.
• SMITH'S locker was also above
GRAY's (though not directly).
• GREEN'S locker was directly below
SMITH'S.
• WILSON'S locker was between that of DAVIS and BOOTH.
• MILLAR'S
locker was on the bottom row directly to
the right of HERD'S.
• WHITE'S
locker was on the bottom right hand corner in the same column as
BOOTH'S.
Which box belonged to Mr. Mathurs?

Ans: Box number 9 belongs
to Mr.Mathurs.


310. Problem based on sets. 100 ppl. 85 are married,
70 have phone, 75 have house, 60 have car. find ppl having house, car, phone and
r married?? easy set problem. i am not sure abt figures but find total ppl who
do not have these things
and subtract from 100 to get ppl having all
these.
ans. 10


311. village and town. in between a hill. a person
travels on cycle 8 km uphill and 24 downhill to reach town continuously in 2hrs
50 min. then he comes to village in 4 hrs and 30 min. find his speed uphill and
downhill.

ans. uphill 6
downhill 16

312. find wizard's age. his
age is 3 times son's age, his father's age is 40 more than twice his age. total
of their ages is 1240.
ans. 360

313. there r some steps. i come down 7
steps then see a man at bottom. then he comes up and i go down at same speed(my
speed). when 4 steps r remaining for me i find tht man has reached the
top.
for my single step downwards he took 2 steps up. find total
steps.
ans. 22 steps.

314) One person went to market to purchase three
varieties of chocolates. He had purchased 20 items with 20 cents. Fudges are
available at 4 cents each, chaco bars are available at 4 for a penny and gum
pints are available at 2 a penny. How many of each item he had
purchased.

315) There is a five digit number, where the third number is
one higher than the sum of first and second digits. Fourth digit is twice of
fifth and third digit is twice the fourth. Second digit is five more than the
first digit. What is the number

316) A cube, which is painted red on all
its sides, is cut into 27 cubes with three straight cuts. Now how many cubes
have

i) No red face
ii)One red face
iii)Two red faces
iv)Three
red faces

317) It is a typical Tap problem. There are two taps, which are
used to fill the tank and one tap to empty the tank. First tap fill the tank in
10 min., while the second takes quarter of an hour to fill the tank if both are
operated independently. Third tap is capable of emptying the tank in seven and a
half minutes. If all the taps are opened simultaneously (when the tank is empty)
how long (if ever) will it take for the tank to get filled
completely?

318) Ten years before, one is seven years more than the "half
the age" of other. Now the bride’s age is 30 years. And also one is nine-tenths
the age of other. What is groom’s present age.

319) Two guys are tossing
coin with a bet of $1 for each game. After some tosses., one guy earned $3 while
the other won three times. How many games do they play.

320) There is one
8 marks analytical question. Which is the easiest puzzle I had ever seen. There
are in total eight members in the group. Five members are to be selected from
that group. They had given three conditions. We have to answer four
multiple-choice questions which are framed based on the above three conditions.
In almost all the questions, answer can be found out by eliminating the answers
from the choices. Going thru the GRE Barrons analytical section will help. Get
the gist of it., rather than trying to solve more and more.

321) There
are four friends, who are speaking about the pizza, which one of them ate
before. Each one will say one statement and we were given that one of them is
lying. We have to find out who ate the pizza. Easy one.., no need to worry abt.
this.

322) All the above nine puzzles are very easy when compared to this
one. This is difficult not because of logic but because of its grammer. The
sentence structure is too complicate. This question reminded me of puzzle 127 of
sakunthala devi’s Puzzles to Puzzle U.

323. there is a 1 km long wire
plces on x poles . if the no of poles is reduced by 1 then he distance of wire
between each poles
increases 1 2/3.how many poles are there
intially.\

324. clark ,jones,mason, smith are 4 ppl. there are 4
professions druggists,grocer,butcher,policeman.find out who is
who?

1)clark and jones are neighbours and they drive each other to
work.
2)jones earns more than mason
3) the police man earns more than the
druggists and i "think" the grocer.
4) the policeman does not have a
druggists as a neighbour.
5) the butcher walks to work
6)policeman does
not meet the grocer until he arrests him for commiting an offence.

325. A
and B write a test
A says " i got a third of the ques. wrong"
B says " i
got 5 wrong"
together they got three quarters of the questions
correct.
how many did a get correct. (5 m)

326. If a die has 1,6 and
3,4 and 2,5 opposite each other how many such dies can be made.

327.
There are three boxes , In one box Two white balls , In two box 2 black balls In
three box 1 white &1 black
The lables on the boxes are not correct.Then
you have to open one box and to find the colour of the balls in all
boxes.

Solution: Open the box labled black& white If white balls are
there then the box labled with white balls
contain black balls and labled
with black balls contain one black and one white ball and viceversa
if two
black ballsare there.


328). there are containing 5 , 7 , 14 , 16 , 18
, 29 balls of either red or blue in colour.
Some boxes contain only red balls
and others contain only blue . One sales man sold one
box out of them and
then he says " I have the same number of red balls left out as that of blue
".
Which box is the one he solds out ?
Ans : total no of balls = 89 and
(89-29 /2 = 60/2 = 30
and also 14 + 16 = 5 + 7 + 18 = 30

329). A chain
is broken into three pieces of equal lenths containing 3 links each. It is taken
to a backsmith to join into a single continuous one . How many links are to to
be opened to make it ? Ans : 2.

330) when the actual time pass 1hr wall
clock is 10 min behind it when 1 hr is shown by wall clock, table clock shows 10
min ahead of it when table clock shows 1 hr the alarm clock goes 5min behind it,
when alarm clock goes 1 hr wrist watch is 5 min ahead of it assuming that all
clocks are correct with actual time at 12 noon what will be time shown by wrist
watch after 6 hr
ans---5:47:32.5
(n X 60 )50/60 X 70/60 X 55/60 X
65/60

331. complete the following
a. $ * * $ @ * ? ?
# @ @ # # $ ?
? some what simillar like this...but not clear.
b. 1 , 3 , 7 , 13 , 21 , __ ,
43 Ans : 31
c. 1, 3, 9, __ , 16900

332. A girl took part in a (some)
game with many others in a circular closed circuit. After pedaling for several
minutes, he found that 1/3th of the cyclists ahead of her and 3/4th of the
cyclists behind him together formed the total no. of participants. How many were
participating in the race?

333. OF all pets i have, except 2 all are
rabbits
OF all pets i have, except 2 all are fish
OF all pets i have,
except 2 all are cats
How many rabbits, fish and cats are there?

334.
given
carpenter + painter = 1100
painter + electrician =
3200
electrician + plumber = 5100
plumber + mason = 2200
mason + labour
= 3000
labour + painter = 1100
find every person's cash? (i dont know the
exact amount mentioned above.)

335. out of 30 questions, the three
persons A,B & C answered 45 correct answers,
B answered 55% of A, B and C
together answered 25 % more of what A answered.
Find how many answers each
answered?

336. Jim,Bud and sam were rounded up by the police yesterday.
because one
of them was suspected of having robbed the local bank. The three
suspects made
the following statements under intensive
questioning.

Jim: I'm innocent
Bud: I'm innocent
Sam: Bud is the
guilty one.
If only one of the statements turned out to be true, who robbed
the bank?


337) There are two containers on a table. A and B . A is
half full of
wine, while B, which is twice A's size,
is onequarter full of
wine . Both containers are filled with water
and the contents are poured into
a
third container C. What portion of container C's mixture is wine
?

338) A wall clock loses 10 minutes every 1 hour. In 1 hour by the
wall
clock , a table clock gets 10 minutes ahead of it. In 1 hour by the
table clock an alarm clock falls 5
minutes behind it. In 1 hour of the alarm
clock, a wristwatch gets 5 minutes ahead it. At noon, all 4
timepieces were
set correctly. To the nearest minutes, what time will the wrist show when the
correct time is 6
p.m. on the same day ?

339) "You see," said
Mrs.Murphy,"Paddy is now one and one-third times as
old as he was when he
took to drink, and little Jimmy, who was forty months old when paddy took
to
drink is now two years more than half as old as I was when Paddy took to
drink , so when little Jimmy is as old as Paddy was when he took to drink.our
three ages combined will amount to just one hundred years" How old is little
Jimmy?

340)Both the Allens and the Smiths have two young sons under
eleven. The name of the boys whose ages rounded off to the nearest year are all
different are Arthur, Bert, Carl and David . Taking the ages of the boys only to
the nearest year ,
the following staements are true

* Arthur is three
years younger than his brother
* Bert is the oldest
* Carl is half as old
as one of the allen boys
* David is five years older than the younger smith
boy
* the total ages of the boys in each family differ by the same
amount
today as they did five years ago

How old is each boy and what
is each boys family name.

341) In a certain organization there are either
men eligible to serve on a eligible to serve on a newly established commitee of
four. The selection of the members is not an easy matter , however for there are
jealousies and attachements among the candidates which prevents a free choice of
four committeemen, if you were the president of the organization could you
select a committee of four satisfying all these whims?

* Ames will serve
with anybody
* Brown won't serve unless Clayton serves
* Clayton wont
serve with Evans
* Davis wont serve without hughes
* evans will serve with
anybody
* French wont serve with Davis unless Grant serves too, and
wont
serve with Clayton unless Davis also
serves
* Grant wont serve
with both Brown and Clayton and wont serve with
either Ames or Evens
*
Hughes wont serve unless either Brown or French serves and wont
serve with
Clayton unless Grant serves too and wont serve with both
Ames and
Evans


342) An artist has exactly seven paintings --- ,T,U,V,W,X,Y,
and Z -- from which she must choose exactly five
to be in an exhibit. Any
combination is acceptable provided it meets the following conditions:

*
If T is chosen , X cannot be chosen
* If U is chosen , Y must also be
chosen
* If V is chosen , X must also be chosen

1) Which one of hte
following is an aceptable combination of
paintings for inclusion in the
exhibit?

A. T,U,V,X,Y
B. T,U,V,Y,Z
C. T,W,X,Y,Z
D.
U,V,W,Y,Z
E. U,V,W,Z,Y

2) If painting T is chosen to be among the
paintings included int he
exhibit which one of the following cannot be chosen
to be among the
paintings included in the exhibit?

A. U
B. V
C.
W
D. Y
E. Z

3)Which one of the following substitutions can the
artist always make
without violating restrictions affecting the combination
of paintings
given that the painting mentioned first was not, and the
painting
mentioned first was not, and the painting mentioned second
was,
originally going to be chosen ?

A. T replaces V
B. U replaces
Y
C. V replaces X
D. W replaces Y
E. Z replaces W

4) If the
artist chooses painting V to be included among the
paintings in the exhibit,
which one of the following must be true of
that combination of
paintings?

A. T is not chosen
B. Y is not chosen
C. U is
chosen
D. W is chosen
E. Z is chosen

343) Yesterday my mother asked
me to buy some stamps. Stamps are available in 2 paise,7paise,10paise,15paise
and 20paise denominations. For three types of stamps I was asked to buy five of
each. For the other two types of stamps. I was asked to buy six of each.
Unfortunately I forgot which I was supposed to buy five of and which to buy six
of
Luckly my mother had given me the exact money required to buy the stamps ,
Rs. 3.00 and the shopkeeper was able to give me the correct stamps. Which stamps
did I buy?

344)Farmer Jones sold a pair of cows for Rs. 210 , On one he
made a profit of ten percent and on the other he lost ten percent. Altogether he
made a profit of five percent. How many did each cow originally cost
him?

345.Meera was playing with her brother using 55 blocks.She gets
bored playing and starts arranging the blocks
such that the no. of blocks in
each row is one less than that in the lower row. Find how many were there
in
the bottom most row?

346.Rahul took part in a cycling game with many
others in a circular closed circuit. After pedaling for
several minutes, he
found that 1/5th of the cyclists ahead of him and 5/6th of the cyclists behind
him
together formed the total no. of participants. How many were
participating in the race?

347.Tom wants to catch a hare. He is standing
250 yards south from the hare. The hare starts moving due east.
Tom, instead
of moving in the northeast direction,moves in such a way that at every
instant,
he is goingtowards the hare. If speed of tom is one and one-third
times that of the hare, find the distance each
traveled before he caught the
hare.

348.Two people are playing with a pair of dies. Instead of numbers,
the dies have different colors on their
sides. The first person wins if the
same color appears on both the dies and the second person wins if the
colors
are different. The odds of their winning are equal. If the first dice has 5 red
sides and 1 blue
side, find the color(s) on the second one.

349.A
company's director said during the board meeting: " The company's income from
roads will be
sufficient to pay 6% of the entire stock issue, but since we
are paying 7.5% interest on the preferred
stock of Rs.4, 000,000 we are able
to pay only 5% of the common stock". Find the value of the common
stock.

350. Mr. ANYMAN left ANYTOWN by car to attend a wedding at
ANYCITY. He had been driving for exactly two hours
when the car got
punctured. It took his driver exactly ten minutes to change the wheel. In order
to play safe
they covered the remaining distance at a speed of 30 mph.
consequently, Mr. ANYMAN was at wedding half an-
hour behind schedule. Had
the car got the puncture only 30 miles later , I would have been only
FIFTEEN
minutes late he told the driver . How Far is ANYCITY from
ANYTOWN.
Ans: 120 miles

351. Alpha, Beta , gamma, delta and epsilon
are friends and have birthdays on consecutive days though may not
be in
order. Gamma is as many days old to Alpha as Beta is younger to Epsilon. Delta
is two days older
then Epsilon. Gamma’s Birthday is on Wednesday. Tell whose
birthday is when.

Ans:
Alpha: Friday
Beta : Saturday
Gamma:
Wednesday
Delta: Tuesday
Epsilon: Thursday

352.The quarter of the
time from midnight to present time added to the half of the time from the
present to
midnight gives the present time. What is the present
time?

Ans: 9hrs past 36 minutes AM

353. A man is going to a
wedding party. He travels for 2hrs when he gets a puncture. Changing tyres takes
10mins.
The rest of the journey he travels at 30 miles/hr. He reaches 30mins
behind schedule. He thinks to himself that
if the puncture had occurred
30miles later, he would have been only 15mins late.
Find the total distance
traveled by the man

354. After world war II three departments did as
follows First department gave some tanks to 2nd &3rd departments equal
to
the number they are having. Then 2nd department gave some tanks to 1st
& 3rd departments equal to
the number they are having. Then 3rd
department gave some tanks to 2nd &1st departments equal to
the number
they are having. Then each department has 24 tanks. Find the initial number of
tanks of each department?

Ans ; A-39 B-21 C-12

355. A girl 'A'
told to her friend about the size and color of a snake she has seen
in the
beach. It is one of the colors brown/black/green and one of the sizes
35/45/55.

If it were not green or if it were not of length 35 it is
55.
If it were not black or if it were not of length 45 it is 55.
If it
were not black or if it were not of length 35 it is 55.

a) What is the
color of the snake?
b) What is the length of the snake?

Ans: a) brown
b) 55

356.A man was on his way to a marriage in a car with a constant
speed. After 2 hours one of the tier is punctured
and it took 10 minutes to
replace it. After that they traveled with a speed of 30 miles/hr and reached the
marriage
30 minutes late to the scheduled time. The driver told that they
would be late by 15 minutes only if the 10 minutes
was not waste. Find the
distance between the two towns?


357. Three clocks where set to true
time. First run with the exact time. Second slows one minute/day. Third gains
one minute/day. After how many days they will show true time.

358. There
were some containers of quantity 1, 3, 4, 5, 6, 12, 15, 22, 24, 38 liters. Each
was filled with some liquid except one. The liquids are milk, water and oil.
Quantity of each was like this. Water = 2* milk oil = 2* water. Find out which
container was empty and containers filled with milk and oil.

359. Two
travelers, one with 64 barrels of wine, other with 20 barrels of wine. They
don't have enough money to pay duty for the same. First traveler pays 40 francs
and gives his 5 barrels, Second traveler gives his 2 barrels but gets 40 francs
in exchange. What's value of each barrel, and duty for each barrel?

Ans:
Value of each barrel-120 francs, Duty on each-10 francs

360. What is
Ann's relation with her husband's mother's only daughter-in-law's sister's
husband?

Ans: Brother-in-law

361. Some guy holding a glass of wine
in his hand looking around in the room says, "This is same as it was four years
ago, how old are your two kids now?" Other guy says "Three now, Pam had one more
in the meanwhile." Pam says, "If you multiply their ages, answer is 96 and if
you add the ages of first two kids, addition is same as our house number." The
first guy says, "You are very smart but that doesn't tell me their ages." Pam
says, "It's very simple, just think." What are the ages of three
kids?

Ans: 8, 6, 2


362. A motor cyclist participant of a race
says "We drove with the speed of 10 miles an hour one way, but while returning
because of less traffic we drove on the same route with 15 miles per hour." What
was their average speed in the whole journey?

Ans: 12 miles per
hour


363. Given following sequence, find the next term in the
series:

(i) 0, 2, 4, 6, 8, 12, 12, 20, 16, ____ Ans: 12

(ii) 3, 6,
13, 26, 33, 66, ___ Ans: 53

364. Three customers want haircut and a
shave. In a saloon, two barbers operate at same speed. They take quarter of an
hour for the haircut and 5 mins for the shave. How quickly can they finish the
haircut and shave of these three customers?

Ans: 30 minutes

365. A
shopkeeper likes to arrange and rearrange his collection of stamps. He arranges
them sometimes in pair, sometimes in bundle of three, sometimes in bundle of
fours, occasionally in bundle of fives and sixes. Every time he's left with one
stamp in hand after arrangement in bundles. But if he arranges in the bundle of
seven, he's not left with any stamp. How many stamps does a shopkeeper
have?

Ans: 301

366. Three different types of objects in a bucket.
How many times does one need to select object from the bucket to get atleast 3
objects of the same type?

Ans: 7


367). A stamp collector has
the habit to arrange or rearrange the stamps accordingly. while doing this he
some times keeps the stamps in pairs, or in group of 3 or in 4 or in or in 6 and
realises that in any case he is left with 1 stamp and when he arranges them in
groups of 7 no stamps remain. what is the number of stamps he has?

368).
amy while walkin down the street with her daughter, meets her husband's mother's
only duaghter in law's sister's husband. how is the related to her? C het
anaS

369). there are 3 custoners who wants to take a hair cut and shave.
there are 2 barbers who takes one quarter of an hour for a hair cut, and 5
minutes for a shave. both the barbers want to finish off and go quickly to their
homes. in what time can do it.

370). we travelled to a place at the rate
of 10 miles per hour and offcourse returned the same way, but owing to less
traffic at the rate of 15 miles per hour.what was our relative
speed.

371). there are 3 types of apples in a box. what is the number of
apples we should take so that we end up with 3 apples of one kind.

372).
a).3,6,13,26,33,66,_ b).0, 1,2,13 ,6,33 ,12, 63,20, 103,_


373. Each
alphabet A,B.. Z is a constant.A=1,B=2,C=3^2,D=4^9 n so on.Each letter is
assigned a value -the position of that letter raised to the value of preceding
alphabet.(C = 3 ^B,D= 4 ^ C n so on)
Compute the numerical value of
(X-A)(X-B)(X-C)....(X-Y)(X-Z).

374. Mr. T has a wrong weighing pan.One
arm is lengthier than other.1
kilogram on left balances 8 melons on right.1
kilogram on right
balances 2 melons on left.If all melons are equal in
weight,what is
the weight of a single melon?

375. In a game of dice, 2
dice thrown at each turn.The score at each
turn is taken as the product of
number on 2 dices.there were five
turns(rolls).second roll is 1 less than
first n (like this ... the
relationship between third n second,third n
fourth,fourth n fifth )
were given.Find the score in first roll,second
roll,third roll,fourth
roll.(jus giving an idea abt question and don know the
exact
relationships).

PURPLE:these are always poisonous if red
are.
YELLOW:six months in a year they are safe.
GREEN: always safe to
consume if purple are poisonous.
RED:six months in a year poisonous.
The
colors are colors of mushrooms available.AT this time
which one is safe to
consume?


G: I am 22
M = G + 2
H = G - 1
H: I am not the
youngest.
difference between M & H is 3
G is 25.

M: M is
younger than G.
G = 23.
H = G+2.
Mr.G,Mr.M,Mr.H made only one statement
false.

Find the ages of all three.
(I am not sure abt the statements
jus chk it out with others)


376 Matrix problem repeated from prev
paper
there are 3 males A,B,C and 3 females X,Y,W. they played 18 games
of
golf altogether.
1. A scored 94
2. X scored 106
3. Y scored
102.
4. Z scored 100.
5. B and C scored 96 and 98 and don't no who's score
what??
6. A's wife beats C's wife.
7. there are two couples whose sum of
scores is same.
Determine who's wife is who and scores of B and
C.


377. A girl had several dollars with her. she went out for
shopping and spent half of them in shopping mall,being generous she had given 1
dollar to the beggar. After that she wqent for lunch and spent the half of the
remaining and gave 2 dollars as tip to the waiter.Then she went to watch a movie
and spent the half of remaining dollars and
gave autorikshaw-wala 3 dollars.
This left her with only 1 dollar.How many dollars did she had with her at the
beggining.

378. A person says that his son is 5 times as old as his
daughter and his wife is 5 times older than his son and he is twice the age of
his wife . The sum total of all the ages equals the age of the grand mother who
celebrated her 81st birthday today.
How old was his son?

379. A
bargain hunter bought some plates for $ 1.30 from asale on saturday,where price
2 was marked off at each article .On monday she went to return them at regular
prices,and bought some cups and saucers from that much amount of money
only.
the normal price of plate were equal to the price of 'one cup and one
saucer'.In total she bought 16 items more than previous. saucers were only of 3
cents hence she brought 10 saucers more tahn the cups,How many cups and saucers
she bought and at what price?

380. A jeweller prepared a window display
each displaying 3 of the 7 jems at a time . They were
methyst,opal,sapphire,emerald, ruby and garnet.Displayed according to the
following conditions:-
1 A sholud always be displayed on the left window and
D on the right.
2 Ruby should never come with any of D or G.
3.E should
always be with S.
then some 4 questions were asked on this. easy
#1 which
combination is appropriate?
AOS;ADR;AES(ans).
#2 which condition is
correct in the right window?
#3 Ruby can be displayed with following other
two?
#4 S can be displayed with the following other two on left side
window?

381. Racing competition.Participants were from 3
tribes
Sonorean-always says truth
Midorean-alternatively says T and F ,not
with any particular start.
Nororean-always False
A says-1. C obstructed me
at the last moment ,which caused me to losethe race.
2. C always speak
true
3. c is the winner.
B says - 1. A is the winner.
2. c says false
always.
C says- 1.B won the Race
2. I didn't caused any obstruction to A
at the last time.
IDentify the tribes of each.

382. A boss tells 1/6
th of his life in child hood,1/12 of his in youth and 1/7 of his in
bachelor,five years after his election
a son was born whom was died four
years ago at half his final age. find the boss age.
. ANS.:74 ths solution
a/6+a/12+a/7 +5......

383. Two thieves went to the museum to stole the
diamonds first thief stole half of them and while going he took another two and
left. Second, third and fourth did the same and there was zero diamonds at the
end. How many diamonds initially at the beginning?

Similar to Q. No. 193
Stolen Mangoes from Shakuntala Devi - More Puzzles Ans: 79

384. A, B, C
are the husbands and D, E, F are their wives not in that order. They are playing
the Golf following these conditions. D, E, F and B scores are as follows
106,102,100 and 94.A and C scores are 98 and 96 not in that order as their names
are not displayed. Two couples get the same score. B wife beat the A wife list
out the wives names and the scores they got.

Ans:
Hus Wife Score
Total
A F 98 102 200 100 198
B D 94 106 200 106 200
C E 96 100 96 102
198

385.A women with dollar bills goto the shopping he spent half of the
money she had for shopping as she was so kind she gave one dollar to the
beggar.she went to the hotel and spent half of the remaning and she gave 2
dollars to the waiter,the she buy some goods with half of the remaining and she
gave 3 dollars to the receptionist. how much money she had in the
begining?
ans:42
come in the reverse order 1+3=4and twice of it 8+2=10 and
twice of it 20+1=21 and twice of it 42.

386.A conducter in the bus ask
the man how old the boy is.the man replied that my son is five times older than
my daughter and my wife is five times older than my son and i am twice older to
my wife and our ages summed upto my granmother whose age is 81 years.can u tell
me the son age?
ans: 5 years
x+5x+25X+50X=81
81X=81
x=1 therefore
son age is 5 years old

387.find out who is oldest and who is youngest
from the following statements...
a)either A or B r the oldest
b)either C
is the oldest or B is the youngest.
ans: A is the oldest and B is the
Youngest

388.one boy tells three ppl to guess mule color..
number one
says its not blk
number two says its eiterh brown or grey
number 3 says
its brown.
..the boy then says one is atleast lying ans atleast telling
truth...finf mule color
ans grey?

389. 2 men take turns walking and
riding one horse that they share...walking speed
4km/hr..riding speed
12km/hr..one rides for some time and ties horse for the other walking fellow and
continues walking......they keep going on like this alternately ..find time that
the horse rests

390. 7 ppl have holidays on 7 diff days...and they give
conditions like A's hol is 3 days
before B etc etc...we have to find the days
on which they take holiday
5.6 subjects 3 ppl teach 2 each...some crazy
conditions and we gotta find the subject for
each..i cant really recall
clearly so why confuse u guys


391. A Lady (say L) is a
philanthropist. she goes to a restaurent,
orders food and pays half the
amount she has and another doller to a
waiter as tip. she then goes to a
mall, does some purchases and pays
half the amount left and another 2 dollers
to a begger outside. At
last she goes to a book store, takes some books and
pays half the
amount left and another 3 dollers to a begger outside. she
then
checks that she only had a doller left to her. How much money she
had
initially? ANs. 42 $

392. Three couples are playing golf together.
The men are E, B & T while women are M G & H. they play 8 rounds.
M,
G, H & E score 106, 102, 100, 94 respectively. while B & T scored either
96 or 98 as it was unresolved due
to error at scoreboard. When finally
resolved they found that two of the couples scored same. It is given
that-
E's wife scored greater than B's wife. Who's whose wife & how much
the men scored each?

393. A lady buys some plates in 130$ with 2$ off
every item. she then returns the plates for the same amount next day to exchange
them for some cups & saucers. Each saucer costs only 3$ each and the no. of
saucers is 10 more than cups. Altogether she takes 16 more items than before. If
she had to buy only cups, how many of them she could have
been taken home on
the first day?

394. Impressed by admiration of the boy by stranger, the
father
said "My son is five times as old as my daughter and my wife is
five
times as old as my son. I'm double the age of my wife and
my
grandmother is as old as the sum of ages of all of us and she
is
celebrating her 81st birthday." what is the age of the boy?

ANS. 5
yrs

395. A, B & C participate in a race & one of them wins. They
belong to
three communities-M, N,
O. O always speak the truth, N always
lie and M alternate. Each of A,
B & C belongs to one community.

A
SAYS:
1. I would have won the race if C had not interfered me at the last
movement.
2. C always speaks truth.
3. C is the winner.

B
SAYS:
1. A wins the race.
2. C is not a 'N'.

C SAYS:
1. I hadn't
interfered with A at the last movement.
2. B wins the race.

Tell,
who's who?


396. Harry is a friend of Axy and Amy. Two statements are
given about
them.
1. if one of Axy and Amy is oldest then another is
youngest.
2. Either Harry is the oldest or Amy is the youngest.
Who is the
oldest?
ANS. Axy

397.sons age is 5 times daughters.mother is five
times son.father is 5 times wife.
Total of all age is Grandpas who is
celebrating 81st B`day.
Ans:-5 yrs

398) One woman buys plates worth
1.30$ at 2cent discount each plate.Then she exchanged the plates
for sausers
& bowl where one bowl & one sausers costs equal to one plate.no of
sausers which costs 3 cent
is10 more than bowl. no of saucers & bowl is
16 more than no of plates.
Ans:-10Plates

399. "One-sixth of my life",
said my boss, "I spent as a child, next one-twelfth as an old boy, one-seventh
& 5
more years in politics & socialization. This brought me upto when
Jimmy born. Jimmy was elected for the
governer four years ago, when he was
half my present age."
How old is my boss?

ANS.: 84 yrs.
(However, I
overlooked that 'one-seventh' part, & got
the answer 36 - a wrong
answer.)
*******************************************************J
= (B/2)
+ 4
(B/6) + (B/12) + (B/7) + 5 + J = B
=> B =
84
*******************************************************

401.A
Couple decided to travel a north country side .so they decide to travel a
minimum amount on car the first day and the second and subsequent day a distance
of 20 miles .If they travel a total amount of 1080 miles. Find he distance
traveled on the 4th day and the 9 day.

402. A card board of 34 * 14 has
to be attached to a wooden box and a total of 35 pins are to be used on the each
side of the card box. Find the total number of pins used .

403. During a
Pizza buffet where A eats more times 2.4 than B, and B eats 6 times
less than
C.find the leat number of times all the three has to eat.

404. Last Year
my cousin came to my place and we played a game where the loosing one has to
give one choclate to the person who won the game .At the end of the vacation,i.e
the day my cousin was leaving she counted number of games that i won an she
won.At last she gave me a total of 8 choclates even though she won about 12
games.
Find the mumber of games that we played.

405. A tree on first
day grows 1/2 of its size second day 1/3rd of its size on the previous day
similarly than 1/4th and so on.u have to calculate after how many days the tree
will be 100 times of its original size.
ans -198 days

406. three
person are there let A B C one always tell truth one always lie and one
sometimes tell truth
they are standing in straight line the first person who
is seniormost and always tells truth tells in middle A is standing the middle
one says C is in the third position the last one says B is in second position ie
middle

407. A,B,C,Dand E are juniors and F,G,H,I are seniors you have to
make three groups each containg three person such that in each group one senior
is there and some other conditions which i cant recall exactly but was like that
if this person will be in group than this cant be in the same group
.

408. there are 100 teams in a football knockout tournament how many
mathces should be held to get the winner answer is 99 ie one less than the no of
teams bcoz in every match on team goes out

409.A car traveling with
uniform speed. There r 15 poles. A car travel from 1 to 10th pole in 10 seconds.
the poles are equally spaced. then how many seconds it takes to reach the 15th
pole? (4)

410. A boy jump to the river from the bridge. He swim opposite
direction of the stream.
After 1000 yards he noted that his hat was fallen at
the bridge. The he goes to take the hat. He didn’t change his speed.What is the
velocity of stream? (4)

411. Uncles A bought a Hat & Suit for
Rs.15.Aunty B bought a Suit with as much as cost of uncle’s Hat. Then reaming
cost
he bought Dresses. Dresses cost is one rupee more than his hat’s cost.
Then she told him his hat’s hat is 1 and ½ cost of
her hat. They spend equal
amount on their purchase .
a. Then uncle said how much is the cost of
Hat.
b. How much they spend altogether ? (4)


412. There are 17
brown ties,13 red ties, 9 green ties, 5 blue ties and 2 white ties.Then a man
takes a tie.
so, how many times he at least take tie to get the 2 ties In
same colour? (6)

413. Mrs. Barbinger bought some plates on Saturday for
$1.30, when everything was being sold two cents below the regular price. She
exchanged those plates on Monday, at their regular price, for cups &
saucers.
Cost of one plate equals cost of one plate & one saucer. She
returned home with 16 more articles than before. Since, saucers cost only 3
cents each, she bought 10 more saucers than cups.
The puzzle is, how many
cups could she have bought on Saturday, for $1.30?

ANS.: 13
cups
*******************************************************
On
Monday:
cup = 12 cents, saucer = 3 cents, plate = 15 cents
On
Saturday:
cup = 10 cents, saucer = 1 cent, plate = 13
cents
*******************************************************

414. Mr
and Mrs ABC purchase suit and hats for 15 Rs. then from remaining money Mrs. ABC
purchase A dress. She shaid " My dress cost is more than 1 Rs from your hat's
cost.
she also added "if we divide our money and then purchase and cost of my
hat is 3 and 1/2 times yours hat then we had spend equal money"
a. " if that
" condition fallows what is price of his hat?
b. Total amount spend ?
ans.
hat= 6 Rs , Total = 22 Rs. ( not sure)

415. My rack contains 8 Red colour
ties, 13 violate colour ties,10 Blue colour ties, 5 Pink colour ties, 4 green
colour ties. If electricity gone and i want at least two ties of same colour
then how many ties i should take out from my rack?
Ans : 6 ties.

416.
Two trains leaving from two station 50 miles away from each other with costant
speed of 60 miles per hour, approaches towards each other on diffrent tracks. if
lenght of each train is 1/6 mile. when they meet How much time they need to pass
each other totally?
ANS : 10 sec. ( not sure)

417. All handsome, fair
skinned, muscular, lean, employed, and rich men are tall.
All handsome men
are fair skinned.
Some muscular men are handsome.
Some muscular men are
not fair skinned.
All lean men are muscular.
No lean man is
handsome.
No fair skinned man who is not handsome is rich.
All tall men
who are neither fair skinned nor muscular are employed.

1. pramod is not
fair skinned. Which of the following must be true ?
a) pramod is
employed
b) if pramod is muscular, he is neither handsome nor lean
c) if
pramod is tall, he is employed or muscular.
d) if pramod is not employed, he
is muscular.
e) if pramod is tall, he may be muscular or handsome, but not
both.
2. which must be false if the information given is true ?
a) no lean
men are fair skinned.
b) some fair skinned are lean.
c) some rich men are
both fair skinned and muscular.
d) some tall men are neither fair skinned nor
employed
e) some rich men are lean
3. which of the following can be
deduced from the information given ?
a) all rich men are handsome
b) some
rich men are handsome
c) some rich men are employed
d) some rich men are
muscular
e) all rich men are handsome, muscular, or employed
4. which
cannot be shown to be true or false on the basis of the information given
?
I. No fair skinned or muscular man is employed
II. Some muscular men are
fair skinned but not handsome
III. No fair skinned man both handsome and
lean

a) I only b) II only c) III only d) I and II e) II and
III

418 In Mulund, the shoe store is closed every Monday, the boutique is
closed every Tuesday, the grocery store is closed every Thursday and the bank is
open only on Monday, Wednesday and Friday. Everything is closed on
Sunday.
One day A, B, C and D went shopping together, each with a different
place to go. They made the following statements:

A D and I wanted to go
earlier in the week but there wasn’t day when we could both take care of our
errands.
B I did not want to come today but tomorrow I will not be able to do
what I want to do.
C I could have gone yesterday or the day before just as
well as today.
D Either yesterday or tomorrow would have suited
me.

Which place did each person visit ?

419 The Novice hockey
tournaments are on for beginners. Just three teams are in the league, and each
plays the other two teams just once. Only part of the information appears in the
result chart, which is given below.
Team Games Won Lost Tied Goals For Goals
against
A 2 1 0
B 2 1 1 2
C 2

The scoring pattern in the
tournament is as follows:

Two points are awarded to the winning team. In
case of a tie, both teams are awarded one point, so the total points in the
standings should always equal the total number of games played ( since each game
played is counted as one for each of the two participating teams). Of course,
total goals scored for and goals scored against must be the same, since every
goal scored for one team is scored against another.
The games are played in
the following order: Game 1: A Vs B; Game 2: A Vs C; Game B Vs C
Can you
determine the score of each of the above games ?


420 A recent murder
case centered around the six men, clam, flip, gront, herm, mast, and walt. In
one order or another these man were the victim, the murderer, the witness, the
police, the judge, and the hangman. The facts of the case were simple. The
victim had died instantly from the effect of gunshot wound inflicted a shot.
After a lengthy trial the murderer was convicted, sentenced to death, and
hanged.

V Mast knew both the victim and the murderer.
v In court the
judge asked clam his account of the shooting.
V Walt was the last of the six
to see flip alive.
V The police testified that he picked up gront near the
place where the body was found.
V Herm and walt never met.
What role did
each of the following play in this melodrama ?
a) Murderer
b) Victim
c)
Judge
d) Witness

421.A alone can do a work in 6 days B alone can do in
8 days with help of c they finished the work in 3 days.If the agreed sum is 640
what is the share of c.(refer r.s agarwal)

422) A boy goes to school from
his house.on one fourth oh his way to school,
he crosses a machinery station.
And on one third of his way to school, he crosses a Railway station. He crossed
the machinery station at 7:30 and he crosses the Railway station at 7:35. When
does he leave the house & when does he reach the school ? (5M)

423.A
drives a car four times a lap 10,20 30,60 kmph what is the average
speed.

424.speed of boat in still water 10 km,if speed up stream is 24 km
and speed down stream is 16 what is speed of the river.

425.If grand
father age is sum all the three grand childern whos age r in equal interval what
is the age of the grand father?

426.In a grass field if 40 cow could eat
for 40 days.The same grass field can feed 30 cows for 60 days.how long
will
it feed 20 cows?

427) An Eraser, Pencil, Notebook together costs $1.00.
Notebook costs more than the cost of 2 Pencils. 3 Pencil costs more than 4
Erasers. 3 Erasers costs more than a Notebook.
How much does a pencil costs?
(5M)

428) Four persons A,B,C,D were there. All were of different weights.
All Four gave a
statement.Among the four statements only the person who is
lightest in weight of all
others gave a true statement.

A Says : B is
heavier than D.
B Says : A is heavier than C.
C Says : I am heavier than
D.
D Says : C is heavier than B.
Find the lightest & List the persons
in ascending order according to their weights. (5M)

428. A man was
travelling to a place 30 miles away from starting point. he was speeding at 60
miles/hr. but when he came back, his car got breakdown and half an hour was
wasted in reparing that. altogether he took 1 hr for return journey. Find the
avg. speed of the whole journey.

429.I'M NOT VERY SURE REGARDING THIS
QUESTION.
A detective was assigned to generate a code using 4 digits, so that
no one could break it. he knew that if the code starts with 0,5 or 7 it will be
cracked. so how many numbers can be formed using 4 digits.

430. A cow was
standing on a bridge, 5feet away from the middle of the bridge. suddenly a
lightning express with 90 miles/hr was coming towards the bridge from nearest
end of the cow.seeeing this the cow ran towards the express and managed to
escape when the train is one feet away from the bridge. if it would have ran to
opposite direction(ie away from train) it would have been hit the train one ft
away from the end of the bridge. Calculate the length of bridge.

431.
there are 3 towns attacked by 3 dragons-x,y,z. Number of days x attack a town is
equal to number of days y attacking another town. Number of days x attack is
equal to half the square root of number of days z attacking a town.number of
days y attacking the town is twice the square root of z.calculate how much days
the curse of each dragon be.

432. A town have a population of 500000 and
42% of males and 28% of females are married to same town. find the total number
of males

433. A and B came back home after their exam and their father
asked them about the test.
A replied-- 1/3rd of my answers were wrong
B
replied-- 5 of my answers were wrong but together we got 3/4 of answers
right.
How many questions were there for the exam?

434) We are given
100 pieces of a puzzle. If fixing two components together is counted as 1 move (
a component can be one piece or an already fixed set of pieces), how many moves
do we need to fix the entire puzzle.
Ans: 18

435) This problem has
appeared b4. 4 kids from 2 families Gupta and sharma... their names are Praveen,
Pra... blah blah...and some relation... which family and what are their
ages..
Ans: Sinhas - 11(praveen)
Sinhas - 5(lalit)
Gupta -
10(pratap)
Gupta - 7 (rajesh)

436) This was one hell of a long problem
- But it has appeared before.. Two girlz after one guy ( sue, swe and sam i
think)..Sue first asks his house no... he replies with two statements (second is
false) so she goes to the wrong house... blah blah.. same thing happens with the
other gal
What was the house number of sue and sam
Ans: 20 and
24

437) A complex statement - about an aeroplane comming late. "The boy
says if it was 6 hours later, the waiting time would be 1/5th of the time if the
plane had come 2 hours earlier instead. the plane is supposed to come at
midnight
Ans 11.00 a.m

438) There are 4 statements and 4 guys ( Dave,
Gus, someone else and one more someone else) - sorry my memory is rotten! Anyway
now these guys make 4 statements abt. who commited the crime. We need to find
out who did it if (i) all but one are saying false and (ii) all but one are
saying true
Ans: (i)archie( ii) tony

439) A kid goes to a bank with a
checque... Comes back and says he spent 35 cents .. his mom says you have
brought back twice what u ver supposed to get. What happened is the novice bank
clerk gave dollars for cents and vice-versa(quote silly) anway... What was the
actual amount??
Ans: don't know...


440. John had decided to divide
his RS.1000/- for his four children according to their ages. The elder child
should be a RS.20/- extra for each than his younger child . What will be the
share of Mahesh whois the youngest?[3 marks]

441.One side of the
measuring arm was longer than the other side .If 3 pyramid width was placed in
longer side it is equal to 2 cube width in the shorter arm.If 8 pyramid width
was placed in longer arm then it is equal to 6 cube width. Let 1 pyramid width
is equal to 10kg.What is the wgt of cube width?

442. A,B,C,D are four
girls who have 1,2,3,4 apples with them respectively.If E have apples equal to
his sister,F have twice the apples as his sister,G have thrice the apples than
his sister and I have four times the apples than my sister.All together we have
32 apples . A,B,C,D are the sisters of whom and
whom?

443.Andy,Brian,Cedric,Dave are architects ,barber, case worker and
dentist but not in the order.
*Architect will have the letter ‘r’ in his
name.
*Atleast one of the person should have coincidence in the first letter
of their name and their occupation but not all [eg: andy-architect]
* Barber
and dentist share their name by only one letter
What is the occupation of
each person?


444.There are 100 bulbs connected to 100 switches
• 1
to 100 all the switches are put ON.
• Only even numbers of switches are used
ie, ON means OFF and viceversa
• Similarly odd numbers of switches are
done
• Switch number which is divisible by 3 are done similarly
• Switch
number which are divisible by 4 are done similarly
This process is done upto
100 divisibles.
When do all the bulbs are in ON and OFF
condition?[8marks]

445. A man wrote his “will’ accordingly the money was
also shared between his car driver,5 sons and 5 daughters.First he gave one
rupee to his car driver,remaining 1/5 of the money is given to his 1st son
.again he gave one to his car driver and remaining 1/5 of the money is given to
his 2nd son, continuing the process until 5 sons are completed .After that he
remaining money is divided between his 5 daughters. What was total amount of
money?

446. 4 people identified a criminal and their statements are:
#
A: Eyes was blue ,height was tall and he wore a hat & a vest.
# B: Eyes
was dark , height wa short and he wore a hat & a vest
# C: Eyes was green
,height was medium and he wore hat & a tie.
# D: Eyes was grey ,height
was tall and he wore a rain coat and a hat.
Everyone said only one correct
identify other two was untrue . How can be the criminal identified?

4470.
A wall clock was slow by 10minutes . According to the wall clock,a table clock
was 10 minutes ahead of it . According to the table clock an alarm clock was 5
minutes behind and according to the alarm clock wrst watch was 5 minutes
fast.Atnoon all the clocks were adjusted .What will be the time at 6 P.M wrist
watch?

448.Professor Kittredge’s literature seminar includes students
with varied tastes
in poetry.All those in the seminar who enjoy the poetry of
browing also enjoy
the poetry of eliot.Those who enjoy the poetry of eliot
despise the poetry of coleridge.
Some of those who enjoy the poetry of eliot
also enjoy the poetry of Auden.
All of those who enjoy the poetry of
Coleridge also enjoy the poetry of Donne.
Some of those who enjoy the poetry
of Auden also despise the poetry of Coleridge.
All of those who enjoy the
poetry of Donne also enjoy the poetry of Frost.

i.
Miss Garfield
enjoys the poetry of Donne.Which of the following must be true?
(A) she may
or may not enjoy the poetry of coleridge.
(B) She does not enjoy the poetry
of Browing.
(C) She does not enjoy the poetry of eliot.
(D) She enjoys the
poetry of Coleridge.

ii.
Mr.Huxtable enjoys the poetry of Browing.He
may also enjoy any of the following
poets except
(A) Auden.
(B)
Coleridge
(C) Donne
(D) Eliot
(E) Frost

iii.
Miss Inaguchi
enjoys the poetry of Coleridge.Which of the following must be false?
(A) she
does not enjoy the poetry of Auden.
(B) She enjoys the poetry of
Donne.
(C) She enjoys the poetry of Frost.
(D) She does not enjoy the
poetry of Browning.
(E) She may enjoy the poetry of
Eliot.

iv.
Based on the information provided,which of the following
statements concerning
the members of the seminar must be true?
(A) All the
those who enjoy the poetry of eliot also enjoy the poetry of Browning.
(B)
None of those who despise the poetry of Frost enjoy the poetry of Auden.
(C)
Some of those who enjoy the poetry of Auden despise the poetry of
Coleridge.
(D) None of those who enjoy the poetry of Browning despise the
poetry of Donne.
(E) Some of those who enjoys the poetry of Frost despise the
poetry of Donne.


a.Rimmie wears a hat only if goerge wears a
tie.
b.George wears a scarf only if Johnie wears a tie.
c.Vickie wears a
googles only if Rimmie wears a hat.
d,e,f,g,h

some what like this
there are 8 statements,from that we have to determine who wears
what.


449. 8 Kigs and 14 Ligs can do 510 tors of work in10days. 13
Kigs and 6 Ligs
can do 484 tors of work in 12 days.
Then find work done by
Kigs and Ligs individually in tors/hr?

450.There is a 3 digited number.
3rd number is the square root of the 1st digit.
2nd digit is the sum of 1st
and 3rd.And that number is divisible by 2,3,6,7.
What is that
number?

451. A boy is playing a game. He took totally 55 blocks and kept
like placing
some x number on the ground,next one less than that above those
blocks
like that till the topmost one is one,like:

x
x x
x x
x
x x x x
x x x x x
x x x x x x
x x x x x x x
x x x x x x x
x
x x x x x x x x x
x x x x x x x x x x
(They didn’t gave this
pictuire,but my explanation is not clear,that’s I gave u this picture)
the
question is how many blocks are there at the base level?

452. There r 100
nations competing for a world-cup.
The board decided to make Knock-out
series.
How many matches to be played for deciding the world
champion?

453.There is log weighing 30kgs. The log having twice thickness
and twice short as first one will weigh howmuch ??

454. there ia truck
which should reach some place at 11`o clock , if it travels with 30 mph it
reaches i
hour before , if it travles with 20 mph it reaches 1 hour late.
what is the distance it must be travlled
and what is the speed it must
maintain to rech at exact time?
ans: 120 miles and 24 mph

455.There is
a square cabbage patch.He told his sister that i have a larger patch than last
year and hence 211 more cabbages this year. Then how many cabbages I have this
year.?
Ans:106*106=11236

456. there are two colcks one runs 1min/hrs
faster and other 1min/hr slower
when will the two clocks have time time
difference of 1 hr :
ans : 30hrs

457. i take a taxi whose no is 3
digit no. it is not divisible by 2,3,5,7
but divisible by 11 it is the
smallest no possible:
ans : 121




458. A man brought some
watermelons to town and sold them. he sold 1/2 more than 1/2 of what he brought
and e was left with one melon. how many melons did he bring to Town? ans:
3

459. When u reverse the digits of age of father u will get the age of
son. one year ago the age of father was twice that of son's age. what are the
current ages of father and son?
ans: 73 & 37

460. There will be
four friends ,one is doctor, one is lawyer... they are having four cars one
ferrari, corvette .... there were some conditions and we were suppose to find
what is profession of each one and what car they own. this was the one for 8
marks.

461. There is a circular ring in which there are 12 black mice and
one white mice . a cat walks circularly in the ring and eats every 12th mice.
where should the cat start so that the white mice is the last one to be eaten by
cat?
ans: if the cat moves circularly it has to start from the 11th mice
(clockwise) w.r.t. to white one.

462. A farmer grows four types of crops
say W,X,Y & Z . two conditions were given:
1. If the farmer grows crop W
in a year then also grows X that year
2. If the farmer grows crop Z one year
then he never grows crop Y next year
There were 6 choices among which we were
suppose to find one correct one which does not violate the two conditions the
choices were some thing like:( first pair denotes crops grown first year and
second after semicolon represents crops grown next year) W,X ; X,Z (this one is
a valid one)
Ans: I don’t remember the choices but the ans was option
C

463. In a class there are less than 500 students . when it is divided
by 3 it gives a whole number. Similarly when it is divided by 4,5 or 7 gives a
whole number. find the no. of students in the class ans: 420

464. There
are three types of birds A,B & C . A costs 5pounds, B costs 3 Pounds and C
costs 1/3 of a pound. find the no. of A,B &C such that u will get 100 birds
for 100 pounds. (I think we were suppose to find 3 answers since there were 3
rows in the answer) ans: A : 4 B : 18 C: 78

465. There are 5 persons who
have won top five places in an event in Olympics . one of them asks all the five
regarding thier positions, they reply as
a: "i am not the last"
b: "c is
in third place"
c: "E is behind A"
d: "B is in first place"
e: "D is
not the first"
The persons who have won gold and silver have lied find the
positions in order(format: name of first, name of second,..)
ans:
B,D,E,A,C
466. A coffee seller has two types of coffee Brand A costing 5 bits
per pound and Brand B costing 3 bits per pound. he mixes two brands to get a 40
pound mixture. he sold this at 6 bits per pound. the seller gets a profit of 33
1/2 percent. how much he has used Brand A in the mixture? ans: 30
pounds

467. You are given with two identical iron bars. one of them is
magnetized and the other is not. u are suppose to find which one is magnetized.
u are not suppose to use any other thing.
my ans: first time i told that i
will suspend the bars freely. but then they told me that i am not suppose to use
any external help. i took some time and then realized that a magnetic bar in the
middle repels towards the end . i told that place one of the bars horizontal
& then move the other one perpendicularly to it . if it repels towards any
of ends then the horizontal one is magnetized if it attracts then is not. They
were convinced

468: What is the result of
(x-a)*(x-b)*(x-c)*...........*(x-y)*(x-z) ?
my ans: i told that since there
is a term (x-x) the answer is zero. They told ok. finally they asked me whether
i have any questions to them i asked them what is the duration of training and
what is field which i will be working on if i get selected.
Anyone looking
for infi solve Shakuntala devi's 2 books, George summers and Ravi narula this
would me more than enough. most important thing is develop ur logical analysis
skill and try to remember the approach rather than answers. don’t panic in the
interview just be cool and confident u will definitely get through. Best of luck
for all

469) a) 10 1 9 2 8 3 7 4 6 5 5 6 4 7 3 8 2 _ _
b) 2 4 16 512
_
Write the next elements in the series.

No comments:

Post a Comment